Добротред 9338 В конец треда | Веб
В связи с тем, что оп начинайкотреда продолжает слоупочить, я взял на себя смелость создать новый, альтернативный тред для начинающих.

Здесь мы будем не спеша и без фанатизма изучать математику, задавать ответы и получать вопросы, а также культурно общаться на смежные темы. ВНИМАНИЕ! Тред — модерируемый! Я собственноручно и прилежно буду репортить любой пост, являющийся нерелейтедом или содержащий мат и оскорбления, и очень прошу модератора неизменно такие посты удалять. Пожалуйста, воздержитесь от грубостей в этом треде! Если вам такой стиль общения не по душе - скройте этот тред.

Остальным же - добро пожаловать в добротред. И помните: главное — хуявное
2 9342
Всем добра и позитива итт
3 9351
>>9338 (OP)
Привет! У меня высшего мат.образования, поэтому прошу гнилые помидоры не кидать - пишу в треде для начинающих.
У меня такой вопрос. Решил я построить модель для школьной евклидовой геометрии в теории множеств (ZFC). Определяю множество действительных чисел R, строю пространство R^3, определяю множество прямых как множество множеств точек, удовлетворяющих условиям. Но пытаясь определить углы, понимаю, что без тригонометрических функций не обойтись. Это так? Чтобы построить евклидову геометрию в ZFC, нужно сначала построить тригонометрические функции (то есть не тригонометрия логически опирается на геометрию, а геометрия логически опирается на тригонометрию)?
4 9353
>>9351
В R^3, как минимум, нужно ввести скалярное произведение, без него ты углы не определишь никак.
5 9360
>>9351
Мне тоже кажется, что вместо прямых тебе нужно ввести векторное пространство прямые тогда будут парой вектора и точки, а в нем - скалярное произведение.

Насчет того, что на что опирается - это вопрос философский. Ни что ни на что не опирается, оно все просто существует. На что тебе проще и удобнее опирать, на то оно и будет опираться, вот.
6 9365
>>9351
можно ввести метрику и несколько угловых аксиом
7 9366
>>9360

Двачую. Такой подход вроде как называется построение геометрии по Вейлю. Сначала определяй векторное пространство, затем скалярное произведение на нем, затем угол между векторами.

Можно также избрать иной путь -- определять геометрию по Клейну: множество с действующей на нем группой преобразований. В случае евклидовой геометрией в R^3, это будут тройки вещественных чисел, на котором действует евклидова, т.е. аффинная ортогональная группа.

Школьный, он же аксиоматический, он же классический евклидов подход значительно менее удобен в работе. Отсюда и всякая ебанина с неполнотой и попытками вывести один постулат из других.
8 9585
>>9366
А ведь можно прямо с углов и начать? Я, правда, не придумал, как одними углами ввести расстояние, не добавляя его явно отдельным числом - тогда все к обычным полярным координатам сводится.

Или вот еще. Можно вместо точек в качестве исходного понятия взять "пятна", то есть множества точек, а сами точки уже ввести как пределы. Вроде физично получается, можно провести аналогии с реальным миром.

Вот например, возвращаясь к углам: можно в качестве "пятен" взять сектора, ну то есть разбиения пространства (ну, пусть пока будет плоскости). Тогда оно у нас будет определяться двумя числами: количеством секторов и номером сектора, который нас нужен. Например, (2;0) - это значит разбить плоскость на две полуплоскости и взять левую. Тут сразу надо показать, что от способа нумерации ничего не зависит. Прямая (ну, или угол) тогда у нас получится, если мы первое число (количество сеекторов-разбиений) устремим к бесконечности. Правда для того, чтобы получить точку, нужно еще какое-то другое разбиение придумать.

Вот, а ведь еще можно показать, что разные алгоритмы разбиений дают тот же результат. Наверное, получатся несколько классов этих алгоритмов, которые дают разную точность. Ну, то есть наверняка ведь есть такие точки, до которых можно добраться, используя один метод разбиения, но которые не получится достать с каким-нибудь другим разбиением.

Только мне что-то никак не приходит в голову, как нам из прямой получить точку. Может нужно взять два разных разбиения, выбрать в каждом из них по сектору и посмотреть, где они (эти сектора, точнее - их "радиусы") пересекаются? Тогда у нас как раз получится и направление, и расстояние на этом направлении. Но вроде это как-то муторно.
9 10499
Тут кто-то лекции Босса просил.
Вот они.
http://rgho.st/private/8ytVrBpz9/bd7c27b7cba52c55ce62bbbe6f86dac2
10 10507
Дано: абсолютные координаты охулиарда точек в трехмерном пространстве.
Нужно: имея только их творить с точками всяческие изъебства, например хуячить оче сложные фигуры и прочее.
Чому я сюда пишу: первый курс был давно, подскажите, что читать.

Спасибо.
11 10513
>>10507
Демки хочешь писать? Если совсем ничего не помнишь, начни с любого учебника по линейной алгебре (можешь Письменного в качестве справочника подрукой держать), а потом переходи на книжки по тридэ-графики (red book, например).
30-1.jpg338 Кб, 1098x385
12 10729
Почему |x| - б >=|а|/2?
13 10731
>>10729
такая дельта
 .JPG1,4 Мб, 3264x2448
14 11872
проверьте пожалуйста
15 12157
ананасы, подскажите годный сайт для подготовки к огэ по матеше, плес
16 12158
>>12157
Матхпрофи, яндекс-егэ.
17 12175
>>12158
благодарю
 .dib10 Кб, 817x188
18 12210
Какого хуя?
19 12212
>>11872
Извини, но никто это читать не будет. Напиши словами, что тебе надо и что ты делаешь
20 12237
>>12212
Почему? Хочу доказать что два определения эквивалентны, которые на тетради сверху написаны.
21 12529
Помогите решить
найти значение k, при котором угол между векторами a(1,k),b(3,3) равен 45

ab=|a||b|cos45, но что-то выходит квадратное уравнение с D<0;
22 12675
>>12529
плохо считаешь значит, всё там нормально получается.
xyz.png5 Кб, 477x365
23 12677
>>12210
кто-то врот
24 12678
>>12529
находишь скалярное произведение 3+3k через координаты. Затем через модули 3*-/(1+k^2). Тройку нахуй и получаешь 1+k^2=(1+k)^2
и находишь, что k=0
25 12681
>>11872
красивый почерк. Ты тня или анимедаун?
26 12744
>>12681
Серьезно? Спасибо ^_____^
27 12747
Ананасы, сап. Нужна помощь в решении простого интегральчика. Могу ли здесь обратиться за помощью? Я просто ньюфаг на вашей доске.
28 12760
>>12747
maple и/или mathematica? не?
29 12761
А можно для сюръективной функции построить right inverse, не используя аксиому выбора?
30 12762
>>12761
В общем случае - нет. Рассмотрим семейство непустых непересекающихся множеств M = {Mi}. Его объединение обозначим как N. Рассмотрим отображение N->M, которое каждому элементу x сопоставляет множество Mi, из которого взят этот x. Сечение этой сюръекции будет функцией выбора на N.
31 12763
>>12762
*функцией выбора на M
32 12764
>>12762
То есть эквивалентны?
33 12771
>>12764
Да. Утверждение, что каждая сюръекция имеет сечение, - одна из форм аксиомы выбора.
Снимок.JPG10 Кб, 189x56
34 12801
Выручайте, братья. Нужно вычислить предел. Ответ 1. Не выручите меня мне край.
35 12838
>>12801
Домножим и поделим на выражение, сопряженное числителю. В числителе станет разность квадратов, иксы сократятся.
 .png39 Кб, 1360x768
36 12863
ЧТО ДЕЛАТЬ БЛЯДЬ?! С первым понятно, а дальше как нахуй?
37 12881
>>12863
Измерь наклон касательной, а вообще ты тупое быдло.
38 12883
>>12881
Это понятно блядь, но как? Нижний тангенс угла 2.5. Ну и как теперь верхний узнать блджад?
39 12926
>>12883
Наклон прямой линии это изменение в y/изменение в x, если изменение в x=1 то изменение в y=-2.5, так понятно?
40 12937
>>12863
перевести на русский, заебала ваша мова
41 12955
>>12926
Но на первом х = 1 соответствует у = 5, а на втором - у = - 5.
42 12957
>>12937
На рисунках график f(х) и касательная в точке -2. Поставить в соответствие значение f'(-2). То есть как я понял это подставить в уравнение касательной -2. Но на первом рисунке это будет просто 5, хотя ответ 0.
43 12961
>>12957
вроде бы спрашивают чему равна производная в точке x = -2.

посмотри что такое производная в твоей книге.

f - ф-я. f` ее производная.

уравнение прямой: y(x) = kx + b
по графикам прямых найти уравнение:
y(x) = kx + b
k = f`(-2)

в первом графике:
y(x) = 5 => k = 0

в втором:
y(0) = -5
-5 = k 0 + b => b = -5

y(-2) = 0
0 = k (-2) - 5 => k = 5/-2
44 12982
>>12961

> f - ф-я. f` ее производная.


>


> уравнение прямой: y(x) = kx + b


> по графикам прямых найти уравнение:


> y(x) = kx + b


> k = f`(-2)


Но f' это И ЕСТЬ касательная к f, по определению. А ты находишь уже производную от неё, то есть f''.
45 12988
>>12982
на твоей картинке, график касательной нарисован зеленым. я и нашел уравнение этой зеленой линии. k, из этого уравнения и есть f`(-2).
производная, f` - не уравнение касательной.
f` в точке, это уголовой коеф. уравнения касательной.
46 13314
Интересует социальный вопрос о поступлении на математика (очно).
Каков в основном возрастной контингент на факультете и каковы взаимоотношения между студентами и студентами и преподавателями?
Например, если мне 24 года( не медалист, не олимпиадник), не будет ли это слишком поздно для на математика?
В смысле, не будет ли насмешек и предвзятого отношения в мою сторону?
47 13315
Где сейчас находит практическое применение алгебра и теория чисел?
типа-книги.jpg102 Кб, 800x600
48 13318
>>13314
Зависит от того, что ты уже знаешь и куда поступаешь. Вообще, бывают математики-первокурсники даже 30lvl, но они уже, как минимум, почитывали пикрелейтед.

>>13315
Везде. Но - только очень простая. Теоретическая математика ушла гораздо дальше, чем нужно практикам.
49 13331
>>13318

>пикрелейтед


Спасибо! Ещё чуть больше года на подготовку есть, может что-нибудь из "пикрелейтед" успею изучить
-q8fBX2yp9Q.jpg74 Кб, 1280x720
Найти константу 50 13340
Матемач помоги найти константу, уже 100 лет как забыл всю эту помойку, а тут срочно нужно решить
51 13364
>>13331
Тривиум вербяшки-калоедина еще ёбни
http://gen.lib.rus.ec/book/index.php?md5=FA81A6949A2210F2D7FF89B44EDBF9E0
http://gen.lib.rus.ec/book/index.php?md5=E9FABE7DEFCA16677237BF618028C684
На пикриле действительно все годнота, но некоторое слишком старое и не нужно начинающему,
ну и прочитать 3 книги по матану это не лучшее, что ты можешь сделать за год подготовки
Из матана прочитай лучше 1 том Зорича (это вообще калькулус, а не analysis) и лекции Львовского
А потом ебашь алгебру
52 13365
>>13364
Отвратительной совет. Особенно с алгеброй. С чего ты взял, что он хочет ей заниматься? И 2-й том Зорича не стоит пропускать.
53 13374
Если не сложно, может кто-нибудь ещё подсказать пособие по всему школьному курсу математики, чтобы знания освежить
54 13377
>>13374
Написанное на каком уровне? Может, проще школьные учебники перечитать?
1490209437.jpg18 Кб, 604x415
55 13453
Нужно указать наклонную А1О на плоскость АВС. Не выручите меня, братья мне край.
56 13458
>>13374
И. М. Гельфанд, А. Шень: “Алгебра”
57 13459
>>13453
И в чем проблема-то?
58 13460
>>13459
Я не знаю как ее указать.
59 13463
>>13460
Так что именно непонятно-то? Что такое наклонная? Что такое плоскость? Или что? Тебя как партизана допытывать приходится, ей-богу.
60 13474
Подскажите плес, чтобы заниматься исследованием операций, на какую специальность надо поступать, на какой кафедре учиться, на какую магистратуру и аспирантуру идти?
61 13597
>>13377
Нет, в школьных учебниках все очень сильно размазано.
Надо, чтобы годно излагалась теория и были задачи профильного уровня.
62 13598
>>13458

>И. М. Гельфанд, А. Шень: “Алгебра”


Так там геометрии нет
63 13599
>>13598
не нужна))
64 13600
>>13599
А как в нее вкатываться?
65 13601
>>13600

>А. Шень: “Геометрия в задачах”

66 13725
Здравствуйте

посоветуйте что интересного можно почить про операционное исчисление, оригиналы?

в универе данную тему не дали..а самому въезжать сложна
67 13731
>>13725
Маслов Операторные методы
68 13732
>>13731
Спасибо.
14655350435740.png238 Кб, 603x454
69 14143
Господа, подскажите пожалуйста хорошие учебнички по физике за старшие классы-первые курсы. Хочу подвспоминать задачки вроде "насколько поднимется давление в сосуде (размеры) с водой (уровень) если его нагреть с n градусов до m градусов" и всё в таком духе.
Обнял.

мимо человечек
sage 70 14144
>>14143
В /sci
Школа - Ландсберг, Младш курсы - Фейнман/Матвеев/Беркли
PentagonTilings15.svg.png198 Кб, 600x358
71 14229
Зачем в геометрии изучают мозаики и паркеты, что это дает для науки, для практического применения?
72 14230
>>14229
Пол можно выложить
73 14260
Посоветуйте, пожалуйста, элементарный учебник-введение по вариационному исчислению
74 14263
>>14229
Так математика не занимается "практическим применением". Математики изучают всякие прикольные штуки, а потом уже из того, что они наизучали, прикладники берут себе ригодившиеся штуки.
75 14307
>>14230
Облегчить жизнь Равшану.
blob2 Кб, 164x39
76 14320
Научите меня решать показательные уравнения с разными основаниями, например вот такое, я чего то туплю
77 14359
>>14320
логарифмируешь обе части равенства
78 14384
>>14359
Тааа не палучаица
79 14433
>>14384
бля пририсуй логарифт к обеим частям и еби мне мозг.
80 14446
>>14320
Делишь обе части на 2^(5-x)
Получаешь:
1 = 4.5 * (4.5)^(5-x)
1 = 4.5^(6-x)
0 = 6 - х
х = 6
81 14612
Мне посоветовали тут изучать Алгебру Гельфанда-Шеня. Я её скачал, начал решать задачи и у меня возникает вопрос почти до каждой. Как их решать? Авторы не дают каких-то вменяемых пояснений по своим задачам. Я худо-бедно дополз за неделю до 51 и 52 задачи, где степени. И я разрыдался, потому что нихрена там не понимаю. Как возраст Земли мне поможет узнать, сколько цифр в десятичной заиписи числа 2^20? Почему они не могут просто объяснить, какие методы мне применять при решении той или иной задачи? Я в отчаянии. Как вообще работать с этой книгой?
 .png20 Кб, 815x328
82 14616
Какому выражению тождественно данное?
83 14622
>>14612

>Почему они не могут просто объяснить, какие методы мне применять при решении той или иной задачи?


И смысл в этом? Ты должен сам находить эти методы. Если не можешь решить, пропускай и решай другие задачи, потом возвращайся к тому что не смог решить.
84 14624
>>14229
Тащемта паркет -- охуенный метод для решения геометрических задач.
85 14634
>>14612
Ни одна из задачек Гельф-Шеня не является эвристической, есть только задачи-шутки на смекалочку.
Полностью прорешать эту брошюрку за день под силу даже пятикласснику (если не ошибаюсь, там около 400 элементарных упражнений: половина на счет, половина очевидные).
Так что, парень, если у тебя возникают такие серьезные проблемы в освоении такого простого материала, и ты еще и рыдаешь, то лучшим советом будет забить на математику вообще, это не твоё.
Ну а если сдаваться неохота, то возьми себе репетитора. С таким-то умишкой заниматься самообразованием не стоит.
Хороший педагог даже такое дно сможет вытащить.
86 14635
>>14634
Может это ты слишком умный?
87 14636
>>9338 (OP)
Всегда была интересна лестница развития математика.
В школьных программах брали всего и по чуть чуть, а как дела обстоят в специальных заведениях?
Как изучить всю математику?
88 14637
>>14636
а вот и школота подъехала

>Как изучить всю математику?


Никак. Даже если ты будешь заниматься по 2 Грота в день, то тебе все равно не хватит и всей жизни, чтобы хорошо разбираться во всех областях

>В школьных программах брали всего и по чуть-чуть


Нихуя там не брали, школьная математика - это наука древних веков, там нет ничего, что не придумали в древности, кроме ошметков матана
89 14640
>>14637
Комплексные числа в школе таки есть.
90 14641
То, что мы сегодня называем "математикой", было почти целиком выковано в XX веке
школьный предмет "Математика" Алгебра/Геометрия к нормальной математике отношения не имеет
одна только синтетическая геометрия и устный счет, от этого тянет блевать
>>14636

>а как дела обстоят в специальных заведениях


Почти также хуево. только в самых топовых школах (57, 239 и подобн) в матклассах детям дают немножко прикоснуться к некоторым совсем базовым вещам, и всё.
Кому нужно - тот ебонит сам.
IMG0322.JPG335 Кб, 606x860
91 14669
Посоветуйте видеозаписей лекций по теории функций комплексного переменного. Хочется поднять свой анализ с днищеуровня.
92 14670
Господа математики, есть следующая проблема оптимизации.
Система из 6 уравнений с двумя неизвестными/параметрами.
Что бы вы использовали, чтобы найти пару параметров (x,y), которые наилучшим образом с наименьшей погрешностью удовлетворяют все шесть уравнений?

Метод наименьших квадратов вручную писать это пиздец, а на работе матлаб без оптимизейшен тул бокса. С лицензией все строго.

Заранее спасибо за быстрый ответ.
93 14671
>>14670
Поправочка - уравнения линейные.
94 14672
>>14670
Возьму х точеный, выражу через у дроченый. Сам за подстановку, мать за грифики с точками пересечения
95 14673
>>14672
Ты не понял в чем суть.
Прямых будет ШЕСТЬ. И никакого пересечения у них не будет. Общего так точно. Нет решения у этой системы.
Но я уже нашел нормальный метод. Спасибо.
 .png9 Кб, 810x159
96 14765
КАКОГО ХУЯ 6 КОГДА 4?! БЛДЖАД МЕНЯ УЖЕ ЗАЕБАЛИ ЭТИ ДАУНСКИЕ ОШИБКИ ЭТИХ """"""""""""""""""""""""""ЭКСПЕРТОВ ЯНДЕКСА"""""""

Может у кого-то лучшая альтернатива для подготовки есть? Пиздец какой-то.
97 14767
>>14616
Хуйня какая-то. Применяя формулу двойного угла выходит Б, но подставляя 0 и 90 градусов выходит В. Какого хуя?
98 14770
>>14765
cosa cosb - sina sinb =
(cosa cosb - sina sinb) cosb/cosb =
(cosa - sina tgb) cosb

sin a = sqrt(8/9)
cos b = -sqrt(1/5)
cosa - sina tgb =
= -1/3 - 2sqrt8 / 3
= (-1 - 2sqrt8) / 3

3sqrt(5) ((-1 - 2sqrt8) / 3) -sqrt(1/5) =
= 1 + 2sqrt8
~= 1 + 5.7
~= 6.7

Ответ: 6
14921119111510.png33 Кб, 547x862
99 14782
>>9338 (OP)
Перекачусь из соседнего треда, где ответа не получил.

>Аноны, поправьте меня, пожалуйста.


Чую что где-то ебанул ошибку. Может даже самую идиотскую в знаках. Но перепроверка с исследованием други способом ничего не дает(да, ответ другой, 21/10, но тут почему не сошлось?).
Где косяк?

>самофикс - у второго интеграла ограничения от 1 до 0 везде, начиная с первой строчки (а то я кое-где проебался с автозаменой)

100 14787
>>9338 (OP)
У меня очень важный вопрос.
Как переформулировать задачу, чтобы постановка вопроса была правильной?
Бросается игральная кость. Каков коэффициент корреляции между выпадением 5 и 6?
101 14788
>>14787
Бросается игральная кость. Есть ли корреляция между выпадением 5 и 6? Нет.
102 14789
>>14788
Хм, а мне сказали, что коэффициент есть и равен -1/5
103 14790
>>14770

А, понял. А как корень из двух-то посчитать, чтоб не проебаться? Не помнить же его наизусть?
104 14791
>>14790
Пусть ты извлекаешь квадратный корень из числа A. В нашем случае A=2.
1. Примерно прикидываешь, чему равен корень. В нашем случае это 1 с копейками.
2. Принимаешь x1 = 1.
3. x2 = 1/2 (x1 + A/x1) = 1/2(1+2/1) = 1.5 = 3/2
4. x3 = 1/2 (x2 + A/x2) = 1/2(3/2+4/3) = 17/12
5. x4 = 1/2 (x3 + A/x3) = 1/2(17/12+24/17) = 577/408
и т.д., пока не надоест.
тест.PNG81 Кб, 1021x702
105 14809
>>9338 (OP)
объясните школьнику, причем здесь проценты оценок А? скуяли 23%?
106 14832
>>14791
Ну охуеть теперь. А если будет корень из трех или пяти? На каждое число помнить алгоритм что ли блядь?
107 14833
По какому алгоритму (НЕ ПОДСТАВЛЯЯ) можно решить левое и КАК РЕШИТЬ ПРАВОЕ БЛЯДЬ? У меня выходит график где только по две точки можно пересечь горизонтальной.
108 14854
>>14833
Находишь экстремумы по производной, хуле.
109 14865
>>14833
рисуй графики
110 14870
>>14832
Корень из трёх - полагаешь A=3, остальные числа не меняешь.
Корень из пяти - полагаешь A=3, x1=2, остальные числа не меняешь.

Это называется "итерационная формула Герона".
111 14905
>>14854
>>14865
Охуительная история. КОНКРЕТНЕЙ МОЖНО БЛЯДЬ?

>>14870
Что такое А?
112 14907
>>14637

>школота


Ты конечно извини, но ты немного ошибся.
Просто мне на старость лет стало интересно всё, что я проебал.
113 14917
>>14905
Целое число, из которого тебе нужно извлечь квадратный корень. Для извлечения корня ты строишь последовательность приближений по формуле xi+1 = 1/2 (xi + A/xi). Число x1 прикидываешь на основе здравого смысла.
114 14932
Какой математический аппарат надо для понимания и применения p-адических чисел?

И посоветуйте, пожалуйста, что-то по модульной арифметики (разные уровни, да?).
115 15435
Здарова, прошу помочь с темой производная, так как преподша не может нормально объяснить. Я почти ничего не понимаю по этой теме, знаю только формулы, но как их правильно использовать не знаю. Надеюсь, что ответите
fihtengoltz.jpg85 Кб, 600x600
116 15459
>>15435
Читни Фихтенгольца.
117 15476
>>15459
Спасибо, обязательно прочитаю
118 15562
>>15459
Открываю такой первый том "Курса..." и сразу же вижу, что в доказательстве невозможности представления sqrt2 в виде p/q пропускается уточнение, что квадрат четного числа есть число четное, и наоборот, на что обращали внимание другие авторы, приводившие данное доказательство.

Конечно, это мелочь, легко доказываемая сообразительным школьником, но год назад она бы поставила меня в тупик. Хорошо ли это или плохо, я не знаю, ведь с одной стороны, дурак вроде меня может забросить курс, а другой я потренировался бы в доказательствах простых утверждений, в случае чего перечитал бы курс элементарной алгебры или спросил бы у товарища.
119 15593
есть множество M. из множества нужно выделить N равномощных подмножеств, |M_1| = |M_2| = ... = |M_n| и найти функцию из M в n. n - индекс подмножества, M_n.

пример:
пусть элементы M - любые натуральные числа.
N = 2 и функция f(m) : m mod(2)
f(m) = 0 если m четное, или f(m) = 1

но в M может быть больше четных элементов, чем нечетных и потому, полученные подмножества не будут равномощными, |M_1| =/= |M_0|

пусть M как в примере. с чего начать поиск такой функции?
120 15595
>>15593
я понял, что просто с любым множеством чисел M, неполучится найти функцию.

ненужно равномощные. но в M_k не должно быть намного больше элементов чем в M_j.
элементы M, это слова из языка, Дом, Живот, ... некоторые слова, напрмер на букву А (англ.), встречаются чаще, чем другие слова.
зная про частоту слов нчинающихся на разные буквы, можно найти функцию?
sage 121 15613
>>15595
разобрался.
122 15681
>>14907
Я тут мимо, но таки скажу, что ты зря это написал. В данном случае школота - скорее уровень развития, чем возраст. Если пятиклассник задает вопрос, как выучить ВСЮ математику (физику, химию, etc) то ему это простительно, он пока не видел вообще ничего, хоть сколько-нибудь приближенного к математике. Но если человек "на старости лет" задается таким вопросом, то не очень понятно, как он сам смог компьютер включить и капчу ввести.
 .png2 Кб, 332x38
123 15683
КАК ЭТО РЕШИТЬ БЛЯДЬ?! ДОШЕЛ ДО

> Х С ХУЕМ СВЕРХУ * 3 С КВАДРАТИЧЕСКИМ ХУЕМ СВЕРХУ = 1


ДАЛЬШЕ ЧТО БЛЯДЬ НАХУЙ?!


https://www.wolframalpha.com/input/?i=log_3(x)^2+++(2x-5)log_3(x)+++x^2+-5x+4=0

Вольфрам жидует на решения, только ответы выдает
(Автор этого поста был предупрежден.)
124 15685
>>15683
Вероятнее всего аналитически не решается.
125 15696
>>15683
Замени логарифм на y и посчитай дискриминант получившегося квадратного уравнения. Дальше просто всё.
126 15704
>>15683
Заменяешь log_3(x) на y, получаешь такую хуйню y^2+(2x-5)y+x^2-5x+4=0 далее воспринимая заведя за скобки получишь эту хуйню (x+y-4)(x+y-1)=0 и тем самым получишь 2 уравнения которые хуй знает как решать x_1+log_3(x_1)-4=0, x_2+log_3(x_2)-1=0
127 15705
>>15683
Видел тебя на форчане.
128 16077
>>9338 (OP)
Мне еще в /sci в мат треде на просьбу дать учебник физики в котором бы шла и математика посоветовали учебник который обозвали "ПТУшным для даунов", я его давно проебал. Может подскажет кто?
129 16082
>>16077
Высшая математика для физиков и техников.
130 16114
>>15696
Не выйдет никакого квадратного уравнения, иксы же остаются.
131 16209
>>16077
Иродов?
132 16212
>>16114
Относительно у квадратное.
133 16232
>>16077
Савельев?
134 16243
>>16212
И дальше что? С иксами что делать?
135 16255
>>16243
Сумма квадратов же.
(log3(x) + x - 2.5)^2 = 2.25
Получаешь два уравнения:
log3(x) = 4 - x
log3(x) = 1 - x
Слева возрастающая функция, справа убывающая, если подберешь решение, то оно будет единственным. 3 и 1
136 16304
>>16255

> (log3(x) + x - 2.5)^2 = 2.25


Где ты это взял?

Я понимаю еще log3(x)(log3(x) + 2x - 5) = -(x-1)(x-4)

И так можно подбором найти 1 и 3, но во-первых это слабо связано с твоей ерундой и во-вторых все еще неизвестно есть ли другие корни.
137 16306
>>16304
Ты не можешь элементарные преобразования провести?
log3(x)^2 + (2x-5)log3(x) + x^2-5x+4 = 0
log3(x)^2 + 2(x - 2.5)log3(x) + (x - 2.5)^2 - 2.25 = 0
Квадрат суммы.
(log3(x) + (x - 2.5))^2 - 2.25 = 0
138 16307
>>16306
Это КВАДРАТ СУММЫ а не наоборот.

Ну и дальше что? Как ты отсюда берешь

> log3(x) = 4 - x


> log3(x) = 1 - x

139 16309

> При каком наименьшем значении параметра a неравенство ax^2-2x + a ≥ 0 справедливo для всех x∈R?



Я просто посчитал а при нулевом дискриминанте и вышел правильный ответ, но ПОЧЕМУ ЭТО РАБОТАЕТ?
140 16321
>>16307
sqrt(2.25)=1.5
141 16322
>>16309
Если D<=0 то полином >= 0
142 16357
Аноны, очень хелп плез.

Как вы знаете есть задачки коммивояжёра:
Имеем 20 точек и пути между ними. четное кол-во путей из каждой вершины.
Начав из точки А в нее же и вернемся, пройдя все 19 чекпойнтов самым выгодным маршрутом.

Моя задачка: есть 40 точек и пути между ними.
Начинаем в А, требуется пройти любых 29 чекпойнтов самым выгодным маршрутом. 10 вершин должно остаться без внимания.

Куда читать? Где могут быть такие примеры?
диплом строится на этом, если есть фланеры-энтузиасты могу расписать подробнее, но вряд ли оно надо
143 16421
>>16357
метод ветвей и границ гугли
144 16436
>>16421
тогда путь будет состоять из всех вершин т.е. из 40
145 16526
>>16321
Это дает

> log3(x) = 4 - x



А это где ты взял блджад?

> log3(x) = 1 - x



И где там возрастающая, а где убывающая функции тоже ничего не ясно.
 .dib6 Кб, 804x121
146 16528
НАЙТИ КОЛИЧЕСТВО ЭКСТРЕМУМОВ.

НУ ПРОИЗВОДНАЯ ВЫДАЕТ ДЖВА НУЛЯ! А ОТВЕТ 5. КАКОГО ?!
147 16533
>>16526
sqrt(2.25)=+-1.5
148 16535
>>16528
ну у производной пять нулей, всё в порядке
149 16567
Сап, пацаны. Короче я задумал замутить генератор криптоалгоритмов. Все бы ничего, задача довольно простая (по крайней мере в случае с симметричными алгоритмами), но вот с алгоритмами с открытым ключем - я не понимаю как все происходит. В каком направлении надо курить, чтобы это сделать? Суть такова, я хочу сделать генератор, который генерирует случайный криптоалгоритм, у которого на шифрование/расшифровку одного килобайта уходит не менее секунды при нагрузке 1 ядра моего проца (phenome II x6 1095) на 100%. Что учить надо для этого? Просто все современные книги по криптографии - полная хуйня. Они описывают лишь уже существующие криптоалгоритмы и расписывают лишь то, что вот это системы с закрытым ключем, это с открытым и все.
tumblrng727kaDGd1s4fz4bo1500.gif1023 Кб, 500x500
150 16988
Здравствуйте! Помогите, пожалуйста, с заданием.

При регистрации в компьютерной системе каждому пользователю выдаётся пароль, состоящий из 9 символов. Из соображений информационной безопасности каждый пароль должен содержать хотя бы 1 десятичную цифру, как прописные, так и строчные латинские буквы, а также не менее 1 символа из 6-символьного набора: «&», «#», «$», «», «!», «@». В базе данных для хранения сведений о каждом пользователе отведено одинаковое и минимально возможное целое число байт. При этом используют посимвольное кодирование паролей, все символы кодируют одинаковым и минимально возможным количеством бит. Кроме собственно пароля, для каждого пользователя в системе хранятся дополнительные сведения, для чего выделено целое число байт; это число одно и то же для всех пользователей.
Для хранения сведений о 20 пользователях потребовалось 500 байт. Сколько байт выделено для хранения дополнительных сведений об одном пользователе? В ответе запишите только целое число – количество байт.
Примечание. В латинском алфавите 26 букв.
Задание 13 № 11349
Пояснение.
Согласно условию, в пароле могут быть использованы 10 цифр (0..9), 52 буквы (строчные и прописные) и ещё 6 символов из спецнабора, всего 10 + 52 + 6 = 68 символов. Известно, что с помощью N бит можно закодировать 2N различных вариантов. Поскольку 26 < 68 < 27, то для записи каждого из 68 символов необходимо 7 бит.
Для хранения всех 9 символов номера нужно 9
7 = 63 бит, а т. к. для записи используется целое число байт, то берём ближайшее не меньшее значение, кратное восьми, это число 64 = 8 * 8 бит (8 байт).
Для хранения всех сведений об одном пользователе используется 500/20 = 25 байт, следовательно, для хранения дополнительных сведений выделено 25 − 8 = 17 байт.

Ответ: 17.


Я считаю, что в решении напрочь забыли о таких вещах как "не менее 1 цифры, одного из 6 символов" в пароле. Т.е если у нас на каждым из девяти символов может быть любое из 68 чисел, то есть хотя бы один вариант, противоречащий условию - когда все девять символов являются любыми из 52 букв, без цифр и тех 6 символов. И эти варианты в решении бодро причислили к верным( а ведь нам нужен наименьший объём). Я никак не могу понять как считать количество информации в таких случаях, выходит вообще решение олимпиадного уровня.

Заранее спасибо.
tumblrng727kaDGd1s4fz4bo1500.gif1023 Кб, 500x500
150 16988
Здравствуйте! Помогите, пожалуйста, с заданием.

При регистрации в компьютерной системе каждому пользователю выдаётся пароль, состоящий из 9 символов. Из соображений информационной безопасности каждый пароль должен содержать хотя бы 1 десятичную цифру, как прописные, так и строчные латинские буквы, а также не менее 1 символа из 6-символьного набора: «&», «#», «$», «», «!», «@». В базе данных для хранения сведений о каждом пользователе отведено одинаковое и минимально возможное целое число байт. При этом используют посимвольное кодирование паролей, все символы кодируют одинаковым и минимально возможным количеством бит. Кроме собственно пароля, для каждого пользователя в системе хранятся дополнительные сведения, для чего выделено целое число байт; это число одно и то же для всех пользователей.
Для хранения сведений о 20 пользователях потребовалось 500 байт. Сколько байт выделено для хранения дополнительных сведений об одном пользователе? В ответе запишите только целое число – количество байт.
Примечание. В латинском алфавите 26 букв.
Задание 13 № 11349
Пояснение.
Согласно условию, в пароле могут быть использованы 10 цифр (0..9), 52 буквы (строчные и прописные) и ещё 6 символов из спецнабора, всего 10 + 52 + 6 = 68 символов. Известно, что с помощью N бит можно закодировать 2N различных вариантов. Поскольку 26 < 68 < 27, то для записи каждого из 68 символов необходимо 7 бит.
Для хранения всех 9 символов номера нужно 9
7 = 63 бит, а т. к. для записи используется целое число байт, то берём ближайшее не меньшее значение, кратное восьми, это число 64 = 8 * 8 бит (8 байт).
Для хранения всех сведений об одном пользователе используется 500/20 = 25 байт, следовательно, для хранения дополнительных сведений выделено 25 − 8 = 17 байт.

Ответ: 17.


Я считаю, что в решении напрочь забыли о таких вещах как "не менее 1 цифры, одного из 6 символов" в пароле. Т.е если у нас на каждым из девяти символов может быть любое из 68 чисел, то есть хотя бы один вариант, противоречащий условию - когда все девять символов являются любыми из 52 букв, без цифр и тех 6 символов. И эти варианты в решении бодро причислили к верным( а ведь нам нужен наименьший объём). Я никак не могу понять как считать количество информации в таких случаях, выходит вообще решение олимпиадного уровня.

Заранее спасибо.
151 17030

> Найдите медиану упорядоченной выборки первых девяти целых чисел из области значений функции


Что это значит?
152 17129
>>17030
Берёшь мн-во значений функции, берёшь первые 9 целых чисел оттуда, находишь их среднее арифметическое.
 .png2 Кб, 191x36
153 17181
>>17129
Что такое "первые целые числа" и каким образом выходит 0?
154 17229
>>16988
если я правильно понял условие, то ты прав и решение неправильное. Я решал бы так
Начал бы сначала с спец.символов -> С69 = 9!/((9-6)!*6!)
Вот
а потом уже все остальные перечисляешь( ну, тупо умножаешь)
решение не сильно изменится, но ответ будет меньше(каюсь, я не считал циферки)
 .png9 Кб, 804x121
155 17447
ПОМОЩЬ
156 17448
>>17447
Найди производную, приравняй к нулю ... профит
157 17449
>>17448
Понял уже. Долго вы тут.
158 17480
Как на английском называется "финально ограниченная функция" (у зорича так)?
finallly bounded не катитj
159 17483
>>17480
А тебе зачем, если не секрет?
Обычно интересуются обратным переводом, с англ. на русский.
160 17484
>>17483
Я просто не понимаю, что он имеет в виду, это его личный термин. Значит есть ограниченная функция, а есть финально ограниченная...
161 17485
>>17480
Никак не называется, это его оригинальный термин.

>>17484
Пусть f определена на X и принимает значения в Y, M - непустое подмножество X. Символом f(M), как обычно, обозначается множество всех таких y из Y, что существует m из M такое, что f(m)=y.

Пусть Y - метрическое пространство.
Функция f называется ограниченной на множестве M, если f(M) целиком содержится в каком-нибудь шаре в Y.

Пусть B - база множеств в X.
Функция f называется финально ограниченной по базе B, если существует множество M из базы B такое, что f ограничена на M.
162 17488
>>17485

>Символом f(M), как обычно, обозначается множество всех таких y из Y, что существует m из M такое, что f(m)=y.


Юмор какой, ты еще знакосочетания определи.
14754681908970.jpg60 Кб, 604x390
163 17490
164 18069
>>13453
AO же, это восьмой класс или девятый?
Телочка.jpg94 Кб, 766x851
165 18422

>когда открыл для себя введение к книжке и там сказано, что "средний студент прочитает её за 2 дня", а ты читаешь уже месяц.

166 18537
>>18422
Вавилов?
Screenshot2017-05-1819-42-37.png48 Кб, 814x244
167 18544
>>18537
Нет.
168 18601
>>17181

ПОМОГИТЕ
169 18603
КАК
170 18727
>>18603
ПИФАГОР
171 18757
>>18601
Тебя наебали. Медиана это не среднее арифметическое.медиана — это такое число выборки, что ровно половина из элементов выборки больше него, а другая половина меньше него. Нужно упорядочить выборку, затем взять средний элемент.
172 19123
Как? Предел этого логарифма, в отрыве от x2 при x → ∞ равен 0 же? Тогда по какой логике производятся дальнейшие действия?
Сколько ни смотрел свойства бесконечно малых/больших, но в итоге ничему не научился оттуда. В каком порядке вычислять? Где применять те или иные свойства? Дурдом.
173 19126
>>19123
Своди к первому замечательному пределу.
174 19134
>>19123
x^2 log (1 - 3/(x^2 + 4)) ~ -3x^2/(x^2+4)
175 19158
>>19134
ты бы ещё написал -3/1
176 19181
>>18757
Это я знаю, но что такое ПЕРВЫЕ 9 ЦЕЛЫХ ЧИСЕЛ? Что это значит блядь? Ведь я не знаю какие первые числа области значений сами по себе, потому что там икс, соответственно сначала надо подставить под него что-то. Но что? Ну вот я пробовал -4 по 4, 0 по 8, все, что только можно придумать - нихуя не 0, а это ответ.
P70527-092846(1).jpg464 Кб, 2158x1168
177 19200
Помогите, пожалуйста, 2 день не могу решить
178 19202
>>19200
я решил
179 19214
Надо привести квадратичную форму к каноническому виду методом собственных векторов (сначала найти собственные вектора матрицы квадратичной формы). Но характеристический многочлен матрицы имеет корни, в формуле которых есть корень третьей степени (не упрощается). ЧЯДНТ?
180 19216
>>19200
умножь нижнее уравнение на 3, дальше все легко
 .png13 Кб, 611x238
181 19229
КАК?! Скаляры АБ - 10, АД - 24. Так какого тут выходит что их квадраты не 100 и 24^2? Как вообще тут можно квадрат 24 найти? Это столбом что ли унижаться?
182 19379
Анончики, вопрос такой: допустим, дана сфера, на ее поверхности находится некоторое количество точек, каждая точка имеет координаты(x,y,z) и массу. я отмечаю на сфере область, в которой находится сколько-то точек. как мне найти центр масс для этой области?
п.с. найденный центр масс должен находиться на поверхности сферы, не внутри ее
w4ruDD9IXOs.jpg37 Кб, 720x404
183 19384
помогите с матлогикой хоть какими-то задумками пожалуйста. Интересуют два последних номера
184 19386
>>19384
Я сам не шарю, но в предпоследней же просто надо построить 2, 1/2 через данные фигни? игра-конструктор получается.
тебе получается дано отношение меньше, которое в себе содержит равно, так тогда определи через них сложение или там функцию следования S(a)=a+1(наоборот из формулы a>b <=> существует c(a=b+c) и a=/=b), потом определи двойку(например, как единственный элемент R, такой, что S(1)=2, потом каким-то еще трюком введи 1/2, а там уже какую хочешь формулу составляй.
185 19388
>>19379

>п.с. найденный центр масс должен находиться на поверхности сферы, не внутри ее


Если бы так было, можно было бы провести через центр масс касательную к сфере плоскость: все материальные точки а твои точки подходят под это определение окажутся с одной стороны от плоскости, в то время как центр масс - в плоскости. Такое невозможно достаточно ввести нормальную к плоскости ось X с нулем в плоскости и посчитать Х-координату центра масс: она получится положительной.
186 19869
>>19388
Наверное, он имеет в виду, что ему надо найти проекцию центра масс на поверхность сферы.
187 20026
Пацаны, по какому учебнику можно угареть по теории множеств?
188 20213
>>19384

Где ты учишься?

x \in {1/2,2} <=>
not((1+1)x<1 or 1<(1+1)x) or not(x<1+1 or 1+1<x)

-это номер 2.
Что подразумевается под понятием "система"? Просто класс множеств?
189 20214
ну или

x \in {1/2,2} <=>
not(x+x<1 or 1<x+x) or not(x<1+1 or 1+1<x)
190 20215
Первая сверху - нет, не является тождественно истинной, так как контрпример - два натуральных ряда с общим началом.
191 20216
хоть анону (>>19384 ) уже, вероятно, не нужен ответ - но мне интересна задача №3.
192 20335
>>20026
Не нашёл ничего понятнее(для меня, офк) чем metamath-овскую брошюрку.
Есть разные очень солидные тома, но они не давали нужного для моего понимания строгости выкладок.

Расширяя дискуссию: у меня такое чувство, что пруфассистанты особенно нужны тем, кто плохо понимает человеческие объяснения.
193 20355
>>20026
Очевидный Мендельсон.
matrix.png2 Кб, 237x238
194 21071
Допусти, дана матрица с данными на картинке, как её привести к каноническому виду?
195 21084
>>21071
для начала найти собственные числа и собственные вектора.
196 21119
>>9338 (OP)
анон нужна помощь, подскажи какие скачать учебники по математике начиная с 5го класса, в школе страдал хуйней в 8 классе было вроде не плохо , дальше скатился в сраное говно ибо ленился и списывал, в универе только на первом курсе была вышка , кое-как сдал, сейчас хочу восполнить знания , задачи научится решать просто для себя , по геометрии вообще пиздец был можно сказать что двойка. Пока писал от стыда аж вспотел. Аноны выручайте с чего начать, какие книги можно скачать на планшет что бы теорию учить и тут же задачки и примеры решать. Думаю что нужно класса с 5го начинать и дальше до 11. Прошу помощи
197 21120
>>21119
ну дык качай школьные учебники, читай теорию, решай задачи, если что не понятно - иди на ютаб, там полно курсов, если что здесь спрашивай. Я вот тоже начал вышку учить, потихоньку продвигаюсь, вроде пока получается.
198 21126
>>21120
да их там много всяких, можешь подскажешь какие получше
199 21135
>>21126
ну у нас такие были, задачник зеленый, а учебник красный.
https://slovo.ws/urok/algebra/08/007/cover_big.jpg
200 21136
>>21126
а вообще есть канал на ютабе MathTutor, там выбирай нужный плейлист и смотри.
201 21138
>>21126
Можешь взять книги Киселева А.П.
202 21149
>>21136

> MathTutor


добра тебе анон
203 21150
>>21138
на работу приду поищу
204 21277
Проверьте, пожалуйста, мои суждения по следующим задачам.
1) "Построить ограниченное множество вещественных чисел, имеющее ровно три предельные точки. " Я думаю, что треугольник.
2) "Построить компактное множество вещественных чисел, множество предельных точек которого счетно." Думаю, что интервал .
3) "Пусть E''= Е∪Е', где Е' множество предельных точек E. Доказать, что Е'' всегда замкнуто и что E''⊂F, если Е⊂F и F замкнуто." Почему F должно быть замкнутым?
205 21278
>>21277
Треугольник - это подмножество R^2. А тебя просят указать подмножество R.
206 21279
>>21277

>Думаю, что интервал


Интервал не является компактным множеством.
207 21291
>>21279
Но он же открытый.
208 21292
>>21278
Извини, немного не понял, почему подмножество R?
209 21293
>>21292

>ограниченное множество вещественных чисел


>множество


>вещественных чисел

210 21311
>>21293
Теперь понял. Ладно. Какой будет ответ?
Подойдёт множество из трёх точек?
211 21338
>>21291
(a,b) - не является компактом в R
[a,b] - компакт в R
впрочем тут надобно спросить что ты понимаешь под компактностью.
212 21358
>>21338
Не, это я затупил. Перепутал сегмент с интервалом.
213 21359
>>21311
Не подойдёт, потому что у него вообще нет предельных точек.
214 21360
>>21311
а ответ - ну возьми три какие-нибудь последовательности точек отрезка [0;2], которые сходятся соответственно к нулю, к 1 и к 2, и объедини их.
215 21374
>>21360
О, спасибо, кажется начал понимать. Кст, это правда, что вариантов выборки в твоём ответе 2?
216 21375
>>21374
Что?
217 21392
>>21375
2 варинта ответа: объединение последовательности, сходящихся к 0, 2 и к 1 слева и справо.
218 21397
>>21392
Нет. Последовательностей, сходящихся к 0, бесконечно много. Последовательностей, сходящихся к 1 и 2, тоже бесконечно много. Разных вариантов ответа - бесконечно много.
Немного переделанная задача коммивояжёра аноним 219 21452
Здравствуйте
Суть проблемы: мне надо решить задачу коммивояжёра но в отличии от обычного варианта этой задачи в моём случае я могу возвращаться в уже пройденные вершины( гамильтонов цикл не обязателен но если он является самым оптимальным то почему бы и нет)
В конечном варианте я хочу сделать программу которая будет находить оптимальный маршрут для работы курьера ( курьер будет выезжать из пункта выдачи проходить по всем местам и возвращаться обратно)
самое сложное как по мне это то что я не знаю как это правильно загуглить ибо если бы я решал обычную задачу коммивояжёра то у меня уже есть 100500 готовых вариантов которые я тупо могу скопировать и вставить но в моей вариации так не выходит
Задача на теорию множеств 220 22132
Пусть X - пространство всех рациональныъ чисел d(p,q)=|p-q| и E - множество всех рациональных p, таких, что 2<p^2<3. Показать, что E замкнуто и ограничено, но не компактно.
Допустимо ли такое решение: Во-первых, множество E - ограничено, так как если квадрат каждого элемента ограничен, то и сам элемент ограничен. Во-вторых, оно замкнуто, потому что X-пространство, а в нём можно выбрать замкнутый шар в силу d(p,q)=|p-q|.Тип показал, что E замкнуто и ограничено. Теперь перехожу к до-ву, что не компактно. Выберем множество: {1/n+2^(1/4),sqrt(3)|n $ N}. Из него нельзя выбрать конечного подпространства. Следовательно, E - некомпактное множество.
221 22184
Привет, народ.
С чего бы начать изучать математику,человеку который вообще хуй соброжает в ней.
Спасибо
222 22185
>>22184
С теории множеств.
223 22221
>>22184
Оснований Математики, особенно конструктивизма.
224 22228
>>22184
Здесь надо поставить вопрос, а чего ты хочешь.
В любом случае лучше всего начинать с теории множеств.
225 22249
>>22221
Поподробней?
Другой, нетралль
226 22288
>>21452
Используй динамику по подмножествам. Если ты хочешь расширить мн-во А вершиной b, тебе придется сначала обойти некоторое подмн-во А, пройти через новую вершину b, и наконец допройти остаток мн-ва А. При всех хождениях придётся использовать кратчайшие пути между вершинами, которые ты предварительно найдешь Флойдом. Работать будет долго, но зато точно.

Писал на коленке, скорее всего обосрался
227 22319
>>22184
>>22185
>>22221
>>22228
Это такой самотроллинг? Почти одновременные посты
228 22322
Здравствуйте, я тут на днях решил повторить ангем, который ещё на первом курсе был. Почитал учебник Александрова, Геометрия. Так вот, там есть задача.
докажите, если на поверхности с двумя семействами прямолинейных образующих есть прямая, им не принадлежащая, то поверхность - плоскость.
Я как-то засторопился на ней. Пробовал доказать аналитически, но там какая-то куча коэффициентов вылазит. Или же частные случаи получаются. А какое-то ясное полное доказательство не приходит в голову.
Хотел попросить помощи.
229 22574
>>22132
Уже 10 дней прошло.
230 22589
Дяденьки, научите меня евклидовой геометрии.
pQ6AauJtqGw.jpg109 Кб, 493x604
Школьный проект 11 класс 231 22613
Сап, тред, я хочу сделать классный исследовательский проект по математике и/или физике, чтобы он меня чему-то научил и было не стыдно его защищать, сам я ничего придумать не смог (абсолютно не имею идей. то, что обычно предлагается в школе очень ущербно, а то, что я сам придумываю скорее похоже на повторение теории, хоть и не входящей в школьный курс) и прошу помощи и советов у местных мыслителей. С меня как всегда - почёт и уважение.
Я физмат 11 класс, планирую поступать на астронома в УрФУж;имею некоторое представление об мат.анализе и некоторых других сферах математики, но в них моё представление менее серьёзно; плохо дружу с геометрией вообще не помню, когда серьёзно ею занимался, но когда занимался она давалась мне достаточно легко, возможно сейчас всё иначе; больше люблю теоретическую часть деятельности (посчитать, подумать, объяснить почему именно так и тд), но не, когда нужно что-то паять, экспериментировать и тд, хотя это не слишком страшно; с самого детства хотел стать астрономом для меня это задача №1, как стать Хокаге. Больше не могу придумать уточняющих слов, если всё-таки нужно ещё - спрашивайте. Хотел начать проект летом, но там уж как получится.
Заранее благодарю за помощь и всё такое
232 22614
>>22613
математические методы в астрономии, астрофизика, геометрия ночного неба
https://www.youtube.com/watch?v=yxtV2HBnEbo
trwhwrth.png7 Кб, 437x310
Интегралы и дифференциалы 233 22949
Во общем я сейчас изучаю анализ Фурье, и у меня есть несколько вопросов на тему "Интегрирование". Сразу предупреждаю, что бы не бомбили, я закончил только 9-ый класс и я все эти темы изучал самостоятельно, поэтому я могу не знать какие-то нюансы которые объясняют в шкалке. Так вот вопросы:
1) Если идёт сложение разных интегралов, то можно не писать dx по сто раз, а вынести его за скобку? Если нельзя, то почему?
2) Я знаю как интегрировать по частям и как преобразовывать дифференциалы, можете посоветовать ещё способов для интегрирования?
3) Ну ещё можете покидать ссылки на полезные ресурсы на эту тему, я просто ещё не до конца по настоящему понимаю значение дифференциала dx.

Заранее спасибо.
234 23002
>>22949
dx показывает, по какой переменной ты интегрируешь. Например, Int xy dx = x^2 y / 2, а Int xy dy = x y^2 / 2. Улавливаешь? И да, выносить за скобки нельзя, это составная часть интеграла.
235 23007
>>23002
Помню в школе сказал, что dx это просто показатель по какой переменной происходит интегрирование. Надо мной весь класс смеялся, включая учителя, обидно было
236 23017
>>23007
Но ведь ты был прав.
237 23043
>>22949
Дело в том, что интегрируются на самом деле не функции, а дифференциалы. Дифференциал - это некий объект, который можно записать в виде f(x)dx - произведения функции и dx. Тут я должен сказать, что такое dx, т.е. ответить на вопрос 3), но я не буду. Это уже наука, а не просто "правила как интегрировать"

Тем не менее, смотреть на dx просто как на указание о том, по какой переменной интегрируешь, мне кажется, для души вредно.
238 23044
>>23043
чтобы не очень наводить тумана, я всё-таки выражусь точнее. Правильно надо было сказать

Дело в том, что интегрируются на самом деле не функции, а дифференциальные формы
239 23060
>>22949

>3) Ну ещё можете покидать ссылки на полезные ресурсы на эту тему, я просто ещё не до конца по настоящему понимаю значение дифференциала dx.


d это корень из 0.
240 23065
Аноны хелп! Какая есть теория по прямой, проходящей через центр вписанной в треугольник окружности и параллельной его стороне(треугольника)?
241 23077
>>23060
Да я слышал про это, что "d это корень из 0, но d неравно 0". Это кажется Ферма придумал.
Я имел ввиду вдруг кто-то знает отличный учебник на эту тему, без воды.
242 23216
Посоветуйте книгу по математике для первого студака первого курса с задачами офк. Закончил высшую шарагу как инженер, а по матеше знаний хуй да нихуя, хотя в начальной-средней школе по матеше чето дрочил.
243 23217
>>23216
С. И. Шварцбурд, О. С. Ивашев-Мусатов. Алгебра и начала
анализа. Учебное пособие для ПТУ.
244 23218
>>23217
Спс.
245 23231
Не могу разобраться с тензорами. Тут можно поспрашивать или мне в отдельный тред?
246 23236
>>23231
Спрашивать можно хотя я всё равно не отвечу, но сначала предупреждение: ни в коем случае не пытайся там ничего осознать через координаты ("компоненты").
247 23244
>>23236
Да, я понял, что целью было развить такую запись, при которой можно было бы не привязываться к какой-то определенной координатной системе, потому что вроде как разрабатывалось это для работы с криволинейными пространствами, в которых происходит какой-то леденящий душу пиздец. Но как я понял, все равно вот этот набор верхних и нижних индексов (контр- и ковариантных) описывает преобразования между системами.

Читал вот эту штуку http://www.gptelecom.ru/Articles/tensor.pdf дочитал с трудом страницы до 10ой, а потом сразу же обосрался. Начинал так же читать брошюру от NASA https://ntrs.nasa.gov/archive/nasa/casi.ntrs.nasa.gov/20020083040.pdf примерно с тем же результатом.

Короче даже не знаю, хочется какого-то более "очеловеченного" объяснения что это и как с этим работать. Читал статьи более общего характера, но в них опускаются все эти преобразования, свертка и пр.

Я вот все как-то раньше думал, что если проявить достаточно упорности и потратить время - все станет ясно. Но хер там был. Векторную алгебру хорошо знаю, с матрицами умею работать (хотя не могу сказать, что везде понимаю саму природу операций), а вот с тензорами вообще жопа.
248 23249
>>23244
Не очень понятно, что ты хочешь от этих тензоров? Если у тебя "криволинейные координаты", т.е. тебе нужна дифференциальная геометрия, то хорошая книга J. Lee - Introduction to Smooth Manifolds. Ещё, помнится, у Стернберга в первой главе Лекций по дифф. геом. было хорошо написано. В обоих книгах, помнится, есть и координаты и универсальные свойства.

Если тебе нужно тензорное произведение как алгебраическая операция, то есть множество источников, и всё гуглится. В том числе листочки из вышки, нму.

Может быть, тебе рано ещё тензоры изучать, а надо как раз гладкие многообразие, касательные расслоения, вот это всё
312314342345345.jpg59 Кб, 653x496
249 24116

>поэтому 3 х Г должно оканчиваться на 5.



Не пойму логики, почему вдруг "ПОЭТОМУ" оно должно оканчиваться на 5?
250 24122
>>24116
Потому что сумма разряда единиц 3xГ и перенесённой двойки равна 7.
251 24263
>>23244
Не еби мозг тензорами. Относись к ним пока как к матрицам: вот есть тензор как таблица чисел, можо умножать только ко- на контр-. Забей. Когда будешь теорию Групп изучать - поймешь что такое тензор.
2017-09-03 211653.png3 Кб, 345x153
Maxima 252 24264
Юзаю wxMaxima, точнее, не юзаю, ибо она на любой мой запрос просто перепечатывает мой ввод.
Как заставить её реально дать ответ, а не просто красиво вывести?
15044671763390.png6 Кб, 142x55
253 24308
Добрый вечер, Господа!

Есть Один Предел: lim(n->oo)(((n!)^(1/n))/n)
Интернет говорит, что ответ здесь 1/е, но решения не даёт. Сам решить не смог.

Желательно не использовать правило Лопиталя, и вообще решить как можно проще. Но рад буду любому решению, и даже совету, кроме совета идти в школу.
254 24328
>>24308
Формулу стирлинга ебани, братка
255 24331
>>24308
потенцируешь по e, затем применяешь совет >>24328
256 24351
>>24308
решил, нет?
257 24438
>>9338 (OP)
Кто нить объясните плзи, почему в совместных вероятностях, когда 2 события формула вычитает их одновременное появление:
А+В=Р(А)+Р(В)-Р(АВ)
а когда 3 и более событий, вероятность когда они появятся все вместе прибавляется
A+B+C=P(A)+P(B)+P(C)-P(AB)-P(AC)-P(BC)+P(ABC)
258 24446
>>24438
есть множества A, B
как найти их |A∪B|?
|A| + |B| - |A∩B|

если множеств больше чем два:
https://en.wikipedia.org/wiki/Inclusion–exclusion_principle
259 24500
>>24438
Формула включений-исключений. Можно по индукции доказать.
260 24517
>>9338 (OP)
такое уравнение
xlog(5/6)=log(0.3)
смотрю решение, там сразу первая строчка
-x
log(5/6)=-1.2039
почему блэд log(0.3) превратился в -1.2039 когда он равен -0.522
261 24518
>>24517
хотя нет там вот так
-xlog(6/5)=-1.2039
но все равно непонятно почему
262 24529
>>24518
Что такое xlog?
263 24530
>>24518
Скорей всего, под log понимается ln. Но если xlog = x log, то это не важно.
264 24532
>>24517
Без указания основания твой вопрос бессмысленный, однако спешу заметить что Ln(0.3) действительно равен -1.2039..
265 24538
>>24532
Не еби мозг: логарифм определен до свободного действия мультипликативной группы.
266 24539
>>24532
короче говоря, когда ты пишешь равенство ты можешь не конкретизировать изоморфизм
267 24542
>>9338 (OP)
Посоветуйте с чего лучше начать изучение математики?
268 24552
>>24542
Со страпона
269 24569
>>24552
проорал
270 24570
>>24542
С матанализа Зорича
271 24571
>>24542
Или с Лорана-Шварца
272 24575
>>24542
С арифметики, серьезно. Возьми учебник Киселев А.П "Арифметика" дабы по его окончанию смог бы сказать, что уверено умеешь делить десятичные дроби и составлять пропорции на пути к алгебре.
273 24582
>>24575
Спасибо,обязательно ознакомлюсь.Как много в день стоит тратить на математику?Или по своему усмотрению?
274 24584
>>24570
Матанализ?Это о чём?Точнее,чему меня научит это?
гетеросексуальный-белый-мужчина.jpg158 Кб, 1200x1600
275 24597
>>24542
Отрасти патлы, но удали лишнюю растительность с тела, сделай маникюр-педикюр, начни лучше ухаживать за собой в общем плане, купи для начала комплект простых тяношмоток, юбочки-чулочки всякие. Нарядись в него, сделай красивую причёску. Всё, ты готов к началу изучения математики.
276 24599
>>24597
Поясни.
277 24604
>>24584
Сколько тебе лет? Какая база есть?
278 24605
>>24597
проорал х2
279 24606
>>24604
>>24604
18 лет.Что подразумеваешь под базой?
280 24609
>>24606
Очевидно, что речь идёт о твоих текущих математических навыках. Ведь если ты задаешь подобные вопросы, то ты, должно быть, капчуешь из глухой деревни, где интернет провели только месяц назад, либо просто не шаришь от слова вообще, потому что знания и книжки тебе были не интересны, и речь не только о математике. Но уточнить не помешает.
Соответственно, начинать читать тут нужно не аспирантские учебники, а что-нибудь более подходящее для твоей ситуации.
281 24613
>>24609
Ну проблема в том,что у нас в школе учили не думать,а просто решать определённые задачи,чтобы мы на ЕГЭ не опозорили школу,и "база" у нас слабая,вот почему я уточнил.Сам без проблем решаю 1 - 14 задачи ЕГЭ.Хочу научиться решать параметры (нас не учили вообще им),понимать и уметь решать все задания,связанные с графиком функции.

>либо просто не шаришь от слова вообще, потому что знания и книжки тебе были не интересны


По правде говоря,тут ты прав.Начал читать,стремиться получать новые знания этим летом.
282 24628
>>24613

>Хочу научиться решать параметры


Берешь сборник задач Сканави и без задней мысли решаешь.
283 24636
>>24628
Хорошо,можешь посоветовать книгу,для изучения функций,графиков,системы координат?
284 24645
>>24636
Математика: Справочные материалы. Гусев В.А., Мордкович А.Г.
285 24651
>>24645
Спасибо,добра тебе.
286 24690
Сап аноны. На первом курсе начали дрочить матрицы. Вроде все действия с ними понятны, но вот в чем беда... ЧЕ ЭТО ВООБЩЕ ТАКОЕ? КАК ОНИ ПОЯВИЛИСЬ? КТО ЭТИ ДЕЙСТВИЯ С НИМИ ПРИДУМАЛ? ПОЧЕМУ ВСЕ ФОРМУЛЫ ДАНЫ БОГОМ? НАХУЯ МНЕ ЭТА ТАБЛИЦА ИЗ ЦИФЕРОК?
Что почитать по этой теме, чтобы прошарить и прочувствовать материал, а не в слепую считать эти определители черточками и треугольничками?
Выручайте!
287 24692
>>24690
https://en.wikipedia.org/wiki/James_Joseph_Sylvester
Читай ссылки в Publications.
288 24693
>>24690
Системы линейных уравнений.
289 24699
Спрошу и у вас.
y''+ (a^2)y = tg(x)
При каких а существует единственное непродолжаемое решение с областью определения (-1;3/2).
Я так понимаю, что раз все функции на этом промежутке непрерывны => мы можем поставить сколько угодно задач Коши и каждая будет решаться. И ответ ни при каких. Мне кажется слишком простое решение, чтобы быть правдой. Да и предыдущая задача решалась точно так же, что меня собственно и настораживает. Все выделенное жирным шрифтом - точное задание, возможно я как-то неправильно понимаю область определения?
20170912013946.jpg3,9 Мб, 4032x3024
290 24700
Можете расписать доказательство в обе стороны и пояснить каждый шаг? Чисто интуитивно это верный пример, но я не знаю, как правильно расписывать доказательство. Дальше идут примеры еще сложнее, поэтому чтобы доказывать их, мне нужно разобраться с дк-вом примера попроще. Заранее извиняюсь за повернутый на 270 градусов пик.
291 24703
>>24690
Смотри как это просто. Есть у тебя матрица A (допустим 3х3) и она умножается на вектор x(3х1): Ax. Это значит, что каждая строчка матрицы как вектор умножается на вектор x (скалярное произведение). Получается 3 строчки - 3 произведения - 3 числа. А теперь тебе нужно найти такой вектор х, что эти скалярные произведения равны каким-то числам. При определенных условиях на А (пусть тебя этим на парах дрочат), такая тройка чисел может быть найдена. То есть мы можем решить уравнение (a_1,x) = b_1 и (a_2,x) = b_2 и (a_3,x) = b_3. Смотри что получилось: мы научились делить в пространстве веторов. Теперь смотри дальше. Есть у нас Ах, но х теперь получается как произведение By: х = Ву. Т.о. Ах = А(Ву). И тут вдруг оказывается, что можно сосчитать сначала (АВ), а потом (АВ)у. Именно по тем правилам, которые ты сейчас учишь, анон.
292 24704
>>24700
Я могу, но я шею сломаю. Выложи нормальную картинку
293 24707
>>24704
Уже разобрался, спасибо
294 24767
Аноны, помогите! Уже неделю мучаюсь, не могу вспомнить.
:Нужна формула расчета следующей ситуации: Каждый год ты откладываешь по 10к на пенсию под 5% годовых. Откладываешь 40 лет. Все.
295 24780
>>24767
a_n = 1.05 a_{n-1} + 10
296 24784
>>24780
a40=1,05{40-1}+10
Я правильно прочитал?
297 24794
>>24784
Не, неправильно. Ты получишь всего 1,5 ляма. Через 40 лет это не деньги. Если таки наш верховный главнокоммандующий как и планирует потратит всю нефть на взятие Литвы, то ты за эти деньги и хлеба на неделю не купишь. Нужно учесть инфляцию и индексацию. Вообще на 40 лет никто не строит прогнозы.
298 24796
>>24794
Эм, к чему ты это все сказал? Мне просто нужна формула расчета таких процентов.
Screenshot2.jpg199 Кб, 1493x383
299 24832
Помогите долбоёбу-первокурснику разобраться с дискреткой, пожалуйста. Есть задания (пикрелейтед). Есть конспект с пары, от которого ничем понятным даже не веет. Как начать шарить в дискретке? Что читать? Может кто-то первые 4 объяснить с примером, а-то даже непонятно, как множества эти строить ориентирванные графы
241.png189 Кб, 1884x680
300 24836
Сап добрач, выручайте,
задача, разрезать гирю на части и положить на края весов так, что-бы они не начали перевешивать в одну из сторон
301 24837
>>24836
l1 m1 = l2 m2 -- условие равновесия.
m1 + m2 = 38
l1 = 2.5
l2 = 4.5
реши систему.
302 24838
>>24836
>>24837
если 40 и 22 вес блюдц, то вместо первого уравнения:
l1 (m1 + 40) = l1 (m2 + 22)

нужно разрезать на целое число частей?

(m1 + m2)/gcd(m1, m2) = 133 части
gcd, это наибольший общий делитель.

если без веса блюдец, то получается меньше частей.
Screen Shot 2017-09-18 at 8.19.10 PM.png63 Кб, 1604x338
303 24991
Пытаюсь вкатиться в матан, правильно ли я решил эти задачи?
1) A + B + C
2) (хз, вроде логично что будет так же как и в первом. но чую что это неправильно)
3) D = (A ∩ B) + (A ∩ C) + (B ∩ C) + (C ∩ A)
304 24992
>>24991

>3) D = (A ∩ B) + (A ∩ C) + (B ∩ C) + (C ∩ A)


бля, здесь поледняя операция лишняя
305 24993
>>24836
разделить 18.5 и на 19.5
306 24998
>>24993
я дебил который забыл что там еще и про условие равновесия
307 25004
>>24998
Простая система.
{100+2.5x = 99 + 4.5y
{x + y = 38

Ответ:
x=170/7
y = 96/7
308 25005
>>25004 адресовалось сюда >>24991
309 25006
>>25005
Сука, блядь.

>>25004 => >>24836
Вот так
1495987158070.jpg51 Кб, 331x331
310 25036
Мне нужна помощь с задачами линейного/нелинейного программирования и выпуклыми множествами/функциями (например, нарисовать геометричечкую интерпретацию условий Куна-Такера или решить задачу методом симплекс). Заданий где-то десятка три, и мне нужно немного объяснить как делать первый пример в них и может быть проверить как я справлюсь с остальными. У меня есть картинки милых одетых девочек и может быть что-то еще.
mathcr`yqabANUScoc_t3kPUNCTUMlYEmi
311 25070
>>24991
первое неправильно
312 25072
>>25070
а что там надо тогда? пересечение всех трех что ли?
313 25073
>>25072
ну подумой. если элемент принадлежит всем, то он принадлежит и тому,..., и тому. Аналог и - пересечение. Алсо, пиши U вместо +, это трушнее
314 25168
Ананас, посоветуй какую-нибудь книжку по теории групп и алгебр Ли для аутистов. Чем проще, тем лучше. В идеале, на русском языке и в понятной для физиков терминологии. С меня как обычно
315 25170
>>25168
Экзамен по стандартной модели в этом семестре чтоли?
316 25176
>>25168
В.Д. Ляховский, А.А. Болохов. Группы симметрии и элементарные частицы
317 25198
>>25170
>>25170
Да нет, просто хочу вкатиться в эту тему. Выглядит интересно
Screenshot3.jpg6 Кб, 349x79
318 25249
Помогите, пожалуйста. В какую сторону воевать? Вообще не идёт в голову ни одно из свойств сочетаний, которое можно было бы пропихнуть для доказательства этого. Что делать? думать
319 25303
>>25249
Такая хуита не доказывается а только выводится.
320 25323
Какая общая формула для количества различных отношений эквивалентности?
321 25326
>>25249
Я не знаю, как это делать, точнее есть идея, но мне лень ее проверять, посчитай c(n+p 2n), а затем придумай подходящую вероятностную задачу и вычисли двумя способами одно и то же число.
323 25430
Привет, матанусы. Посоветуйте какой-то вариант для численного расчёта формул, набранных в латехе
324 25434
Продублирую вопрос и тут, дабы скорее получить ответ
Сап, матач
Подскажи дурачку: как найти точку на окружности, которая лежит на одной прямой с другой точкой, которая находится не на окружности
325 25436
>>25434
Отбой !
Дурачок додумался
326 25645
Ну и сюда напишу.
Ребят, помогите вкатиться в геометрию 7-8, что читать? Что учить? срок - 14 дней да, я школьник
327 25712
>>9338 (OP)
Вообщем есть 1681 количество клеток, и есть две переменных: x со значением 1.012 и y со значением 1.192. Они распределены равномерно по этому полю. И они жрут друг друга ровно на эту разницу за один ход. Нужно с помощью формулы понять, за какое кооличество времени переменная y займет полностью поел сожрав переменную x. По какой формуле это вычислять? Я помню какие-то прогрессии гемоетрические или арифмитечские есть, но это все не то. Дайте хотя бы наводку, по какой формуле все это вычислить, я в матане вообще ноль, если что.

Вот, для наглядности, о чем я

http://www.netlogoweb.org/launch#http://www.netlogoweb.org/assets/modelslib/Sample Models/Biology/Evolution/Altruism.nlogo
328 25837
Сап Антоны
Имеется вопросик по матану про неопределенности(в пределах).
Что это вообще такое? Почему 0/0 - это неопределённость? Ведь мы бесконечно малое число делим на бесконечно малое и, по моей гуманитарной логике, получаю бесконечно малое тобишь 0. Но ведь это не так и от этого просто нужно избавляться, как говорят в вузе. А ещё вот 1 в степени бесконечность? Мне в шкалке говорили, что 1 в ЛЮБОЙ степени - это один. Почему в анализе не так?
Ps. В вузе учат только работать с неопределенностями, но про них ни слова не говорят.
329 25848
>>25837

>Ведь мы бесконечно малое число делим на бесконечно малое и, по моей гуманитарной логике, получаю бесконечно малое тобишь 0.


Рассмотрим предел x/x при x->0, очевидно, что он равен единице, в тоже время твоя "логика" утверждает что это должен быть нуль.
IMG20171015192258.jpg1,7 Мб, 3264x1836
330 25893
помоги решить
331 25900
>>25837

>бесконечно малое число


В этом твоя проблема. Не бывает "бесконечно малых чисел".

Фраза "бесконечно малое число делим на бесконечно малое" по сути означает "берем две бесконечно малые последовательности/функции (т. е. с пределом, равным 0), и делим одну на другую". В высшей математике 0 на 0 (числа) делить тоже нельзя. Можно делить бесконечно малые последовательности или функции, которые просто для краткости обозначают их пределом, то есть нулем. И правила для такого деления только отчасти совпадают с правилами деления для чисел: аналогия есть, а идентичности нет. И все они доказываются отдельно.

Аналогично с 1 в степени ∞. Имеется в виду последовательность/функция с пределом 1 в степени последовательности/функции с пределом ∞ (т. е. расходящейся в бесконечность). Дальше все как с 0/0: это условность, похожая по многим свойствам на возведение в степень чисел, но им не являющаяся.
1508091355928.jpg561 Кб, 2090x2091
332 25905
Без названия45.png1 Кб, 241x21
333 25966
Помогите пожалуйста(немного запутался тк я тупенький :3)
334 25967
>>25966
Шестой класс?
KXaD9PGz9xY.jpg540 Кб, 1536x2048
335 25972
сап, аноны, объясните что делать с синусом в первом задании, с натуральным логарифмом во втором, ну, и если не трудно, то решите 3-е и 4-ое. буду премного благодарен
336 25974
>>25972
|sin| ≤ 1, улавливаешь?
337 25975
>>25974
ты намекаешь на то, что это один из тех случаев, когда можно тупо подставить число (в данном случае 1) вместо x?
338 25976
>>25975
Я намекаю на то, что если f ≤ g, и пределы существуют, то lim f ≤ lim g.
339 25977
>>25976
а можно теперь простым языком?
340 25978
>>25977
Блядь, да, можно заменить синус на 1. Только пока ты не поймешь, почему можно, так и будешь просить решить элементарную хуету на двачах.
341 25979
>>25978
я бы не прочь понять, но мои знания в этой области столь поверхностны, что я не понимаю, где в данном примере эти пресловутые f и g
342 25980
>>25979
Обозначим f = sin, g = 1. f <= g, поэтому lim f <= lim g = 1 (если lim f вообще существует, правда, но в данном случае это не важно). Поэтому, lim sin * h <= lim h для h >= 0. Осталось показать, что lim h = 0 (надеюсь, ты уже догадался, что нужно обозначить за h, а также, как доказать, что ответ еще и >= 0 аналогично).
343 25981
>>25980
И да, расписывать всю эту байду не нужно. Ее надо понять.
344 25982
>>25980

>lim h = 0


если здесь нет опечатки то не понял...
Screenshot (7).png27 Кб, 572x147
345 25984
Призываю мамкиных математиков двоща. Хелпаните первокурснику.
346 25997
>>25984
Не решил еще? Задача скорее на свойства логарифмов, чем на пределы. Вынеси квадрат. Затем верхний логирифм разбей на сумму двух. Потом подели почленно. Одна часть стремится к 0. Вторая - константа 1/ln10. У нас еще остался квадрат, надо в него возвести.
347 26000
>>25997
Уже решил, но спасибо.
348 26003
>>25972
Я присоединяюсь. Ну 1 вроде просто, если пользоваться оценкой и эквивалентностью. Второе не смотрел особо. В третьем через эквивалентные убрал логарифм и кое-как перегруппировал знаменатель, но дальше что делать не знаю. В четвертом по асимптотикам раскладываем и упрощаем дробь до вида (1 + a/x), получаем второй зам. предел.
349 26091
Сап, /math/. Как определить количество номеров машин, содержащих две и только две одинаковые цифры (номер машины четырехзначный 0000..9999)?
350 26142
>>26091
ВСЕГО - БЕЗ_ПОВТОРОВ - 3_ОДИНАКОВЫХ - ВСЕ_ОДИНАКОВЫЕ
10^4 - 10 9 8 7 - 4 10 * 9 - 10
351 26143
>>26142
Хотя проще так, наверное:
3 x 2 x 10 x 9 x 8
352 26171
>>26142
>>26143
Тогда уж ещё 2 пары одинаковых
353 26175
>>26091
10 (первая цирфра любым способом) x 1 (вторая повторяет первую) x 9 (третья любая кроме первой/второй) x 9 (то же самое) x 4! (перестановки)
354 26180
Нужны умнейшие! Применение математики в железнодорожном транспорте, дайте статью плеееес
355 26187
>>26171
Да, проебался в первом варианте.
36899168-Math-vector-seamless-texture-with-algebra-plots-tr[...].jpg173 Кб, 1300x659
356 26218
>>26187
Привет, математик
Слушайте, хочу начать сечь в математике. Для начала - самой обычной, школьной программе. Дело в том, что я был распиздяем, ничего не учил, сдавал экзамены по алгебре/геометрии на тройки только потому, что сосал хуй преподавателя. Последнее, что помню из школьной программы - это как найти Хуй преподавателя, и то смутно, это какой класс, шестой?
У меня уже полгода постепенно растёт в башке навязчивая мысль начать вкатываться в математику, просто, блядь, неумолимо хочется, и я не знаю, почему. Беспричинно. Нужна помощь советом, анон. Если я начну курить обычные школьные учебники, это будет нормально, чтобы постичь, собственно, школьную программу? Каких авторов учебники предпочесть, кто толковее пишет? Или, может, имеет смысл изучать это дело по каким-то другим методикам? На такие случаи жизни есть ссылки в вашем пастебине?
То есть, для совсем отсталых в этом плане людей, типа меня. Подскажи, анон.
357 26219
>>26180
Ну как минимум планирование расписаний
359 26237
>>26218
Да школьные учебники самое то. Они все примерно на одном уровне, скачай несколько, выбери те, которые больше понравятся. Главное делай упражнения, простое чтение тебе ничего не даст. Можешь ещё обратить внимание на khan academy, но это лучше в качестве дополнения.
360 26416
Анон, хочу укрепить свою математическую базу, т.к. не абсолютно не умею доказывать теоремы. Решил начать практически с нуля, класса так с шестого. Стратегия моя весьма проста: дрочить листочки и олимпиадные задачки. Задачки найти не проблема, а вот с листочками беда. Не подскажете где можно достать? Дальше-то НМУ-шные пойдут, а мне бы школьного уровня. Ну или учебники хотя бы для продвинутой школоты, где все строится на определениях и ворохом "докажите что" к ним.
361 26419
>>26416
>>26237
>>26218
Зачем вам учебники, если есть книги типа "алгебры" гельфанда и шеня, гашкова, алексеева?
362 26420
>>26419
Спасибо, анон. Я как раз спрашивал за подобные книги, т.к. не знаю про литературу школьного уровня абсолютно ничего. Если есть еще что близкое к листочкам, то милости прошу.
363 26422
>>26419
Это книги не для нулевых.
364 26438
>>26422
Ну да. Надо уметь складывать в столбик и знать таблицу умножения. Остальное там объясняется, впрочем.
365 26445
>>26438
9/10 с таким набором знаний застрянут на одной из первых задачек в Гельфанде, не говоря уже про остальные две книги.
366 26446
>>26445

>на одной из первых задачек


Типа "умножьте 57 на 10101010101"? Ты эту книгу открывал вообще?
Более того, я тебе скажу что гораздо проще застрять на задаче со звёздочкой в каком-нибудь учебнике Макарычева за седьмой класс. Там кстати дофига задач на доказательство.
Учебников хороших нет вообще, Колмогоров это полная хуйня, у него научишься разве что наивной теории множеств. Лучшее что я видел это Виленкин-Шварцбурд для матклассов, там хоть есть про кольцо многочленов и основную теорему алгебры, но опять же без внятноно определения тех же колец. Учебники это просто кал по умолчанию, лучше не тратить время на них. В крайнем случае есть справочник со всеми школьными фактами и формулами, типа Аленицын-Бутиков, там всего 300 страниц, еще и по физике вся программа, не только математика.
367 26485
>>26416
Специально для маленьких и пушистых https://www.people.vcu.edu/~rhammack/BookOfProof/
Небольшой ликбез про доказательства.
>>26445
Кстати, сдюжил пруф к задаче про ряд Фарея. Не без подсказки, но в целом самостоятельно.

мимо brainlet, который затупил на №44.3
368 26487
>>26485
Интересная задача. Две дроби a/b и c/d для натуральных a, b, c, d соседние, если ad - bc = |1|.
Доказать, что не существует дроби e/f с опять же натуральными e, f, такой что a/b < e/f < c/d; f < b+d.
Всё условие правильно понял?
Пусть a = 2, b = 3, c = 5, d = 7, e = 6, f = 9.
Двп умножить на семь четырнадцать, три на пять пятнадцать, разность 14 - 15 = - 1. Значит дроби соседние.
f = 9 < 3 + 7 = 10, тоже подходит.
Но 6/9 больше чем 2/3 и меньше чем 5/7 (пруф на калькуляторе), что дает контрпример и показывает ложность утверждения.
мимо подумал две минуты
369 26488
>>26487

>6/9 больше чем 2/3


>дает контрпример


Пиздец.
370 26489
>>26488
Двачую. Лучше бы ряды Фарадея посчитал сперва и убедился что множество дробей с натуральными основаниями всюду плотно!
371 26491
>>23077>>22949
Учебник-то есть, конечно, и не один. Английский хорошо знаешь или тебе на русском только?
372 26695
Нужно научиться строить сечения многогранников, где?
373 27560
6 и 5 являются взаимно простыми или же нет?
374 27562
>>27560
Да. Сука, определени загуглить не мог?
123.jpg76 Кб, 790x297
375 27563
>>27562
Уточнить зашел. Вкатываюсь потихоньку и блять простейшую хуйню понять не могу.
Но уже вроде разобрался.
376 27567
Как посчитать Binomial[-1,-k] k>0? k целое.
377 27627
>>27567
-1 + k
Бином по определению двухчлен. Два члена "-1" и "k".
Каждый многочлен(тут двухчлен) объединяет одночлены со знаком "+".
Одночлен, может содержать все, кроме операции "+"и "-".
- = +- . Операцию "-" можно заменить "+-". Минус может считаться частью числа.

-1+ k = k - 1, так как +(-1) + k =k +(-1). Коммутативный закон.
a + b = b + a a + (-b) = (-b) + a
1 + 2 = 2 + 1;
1 + (-1) = 1 - 1 = 0
-1 + 1 = 1-1 = 0

Аргумент "k" может принимать любое целое число.
Y = k - 1
Если "k" = 1, то Y = 0, так как Y= k -1 = 1 -1 = 0

Чтобы вычислить полином с целым числом k, нужно от него отнять единицу.
378 27756
Если у нас есть отображение из f: N в N , которое каждому n ставит в соответствие n^2, то почему для такого отображения не существует правого обратного?
379 27780
>>27756
Что такое ''правое'' обратное?
photo2017-11-1420-16-47.jpg145 Кб, 560x517
380 27810
Это легкая физика, матач, помоги, пожалуйста.
BC98F1EE-7DD9-41E9-91C5-9EEC6E4B829D.png1,3 Мб, 2048x1536
381 27969
Как это следует-то?
382 28045
>>27560
>>27969
Проиграл с разницы.
383 28129
>>28045
Ну такэто же ОБЗОР, вот и куча примеров где нетрудно увидеть, очевидно, легко понять что.
Помню как Хеллер рыдал от этой книги.
56af6736ea0801529d2b2eaf.png204 Кб, 500x446
384 28192
Привет, народ.

Вот пзд нулевый в математике, даже таблицу умножения плохо знаю.
А хочу жутко хотябы бузу знать, но не знаю с чего начать*
Можете дать четкую последовательность изучение великой науки)
Пример:
1.Арифметика
2.Таблица умножения
385 28196
>>28192
Хорошо.
1. Линейная алгебра: Модули, гомоморфизмы, точная последовательность, подмодули и фактор-модули. Свободные модули, базис, размерность. Hom, категория модулей, алгебра эндоморфизмов.
2. Теория Галуа: Тензорное произведение, внешняя степень, определители. Конечные расширения полей как ассоциативные алгебры, полупростые модули, когомологии циклических групп, символ норменного вычета через автоморфизм Фробениуса, теория полей классов, вычисление K(2) поля, плюс-конструкция Квиллена.

Пока хватит. Как освоишь, приходи.
386 28197

>>>28192


Обратись к школьной программе.
387 28226
>>28192
Книжка "Алгебра" Шеня.
388 28237
>>28192
Все ищешь в интернете. Учи, все время повторяя.

0.Учишь названия составных частей при добавлении чисел (слагаемое, сумма)
1.Таблица суммы(таблицу плюсования) однозначных чисел
2.Понятие "разряда" числа[десятки, сотни, тысячи]
3. Понятие "класса" числа[тысячи, миллионы, миллиарды и тд]
4.Правила добавления чисел между разрядами.
5. Суммирование в столбик.
ТЕПЕРЬ, ТЫ УМЕЕШЬ ДОБАВЛЯТЬ ЛЮБЫЕ ЧИСЛА ДРУГ К ДРУГУ. ПРАКТИКУЙСЯ. Старайся все делать устно, а не в столбик.

0.Учишь названия составных частей при умножении (множитель, произведение)
1.Таблица умножение(2*3=6)
2.Правило умножение однозначных чисел на двухзначные
3.Правила умножение однозначных на те, которые имеют больший разряд.
4. Правила умножения двухзначного на двухзначное
5.Правило умножение вне зависимости от разрядов.
6.Умножение в столбик
Теперь, ты умеешь умножать. Практикуйся. Старайся без столбика

0.Учишь составные при отрицании. (Уменьшаемое, отнимаемое, разница)
1.Учишь таблицу разницы(знак "-") однозначных чисел
2.Решаешь множество примеров делений, обучаясь таблице деления обратной таблице умножения.
3.Учишь составные при делении(делимое, делитель, частное)

1.Изучаешь понятие кратности на 2,3,6,9,10
2.Изучаешь деление в столбик

1.Изучаешь понятие " отрицательных" чисел(запоминаешь понятие "целых" чисел в отличии от "натуральных")
2.Учишь и практикуешь правила суммирования и отнимании
3.Учишь правило определение знака при умножении

1.Изучаешь основы понятия "дроби", а также термины(числитель, знаменатель, отношение, рациональное число)
2.Изучаешь понятие "простое число"
3.Практикуешься разлагать сложные числа на простые.
4.Учишься находить общий знаменатель.
5.Учишься сокращать дроби
6.Учишься добавлять дроби
7.Учишься умножать дроби
8.Учишься делить дроби
9.Учишься обнимать дроби

Напишешь, когда будешь готов. Ты уже должен будешь хорошо считать целые числа, дроби с любыми операциями.

Дальше можно изучать линейные уравнения, графики функций, квадратные уравнение... Основы геометрии, типа углов, отрезков, прямой, плоскости.

Ищешь, школьные учебники и читаешь параллельно в интернете.
388 28237
>>28192
Все ищешь в интернете. Учи, все время повторяя.

0.Учишь названия составных частей при добавлении чисел (слагаемое, сумма)
1.Таблица суммы(таблицу плюсования) однозначных чисел
2.Понятие "разряда" числа[десятки, сотни, тысячи]
3. Понятие "класса" числа[тысячи, миллионы, миллиарды и тд]
4.Правила добавления чисел между разрядами.
5. Суммирование в столбик.
ТЕПЕРЬ, ТЫ УМЕЕШЬ ДОБАВЛЯТЬ ЛЮБЫЕ ЧИСЛА ДРУГ К ДРУГУ. ПРАКТИКУЙСЯ. Старайся все делать устно, а не в столбик.

0.Учишь названия составных частей при умножении (множитель, произведение)
1.Таблица умножение(2*3=6)
2.Правило умножение однозначных чисел на двухзначные
3.Правила умножение однозначных на те, которые имеют больший разряд.
4. Правила умножения двухзначного на двухзначное
5.Правило умножение вне зависимости от разрядов.
6.Умножение в столбик
Теперь, ты умеешь умножать. Практикуйся. Старайся без столбика

0.Учишь составные при отрицании. (Уменьшаемое, отнимаемое, разница)
1.Учишь таблицу разницы(знак "-") однозначных чисел
2.Решаешь множество примеров делений, обучаясь таблице деления обратной таблице умножения.
3.Учишь составные при делении(делимое, делитель, частное)

1.Изучаешь понятие кратности на 2,3,6,9,10
2.Изучаешь деление в столбик

1.Изучаешь понятие " отрицательных" чисел(запоминаешь понятие "целых" чисел в отличии от "натуральных")
2.Учишь и практикуешь правила суммирования и отнимании
3.Учишь правило определение знака при умножении

1.Изучаешь основы понятия "дроби", а также термины(числитель, знаменатель, отношение, рациональное число)
2.Изучаешь понятие "простое число"
3.Практикуешься разлагать сложные числа на простые.
4.Учишься находить общий знаменатель.
5.Учишься сокращать дроби
6.Учишься добавлять дроби
7.Учишься умножать дроби
8.Учишься делить дроби
9.Учишься обнимать дроби

Напишешь, когда будешь готов. Ты уже должен будешь хорошо считать целые числа, дроби с любыми операциями.

Дальше можно изучать линейные уравнения, графики функций, квадратные уравнение... Основы геометрии, типа углов, отрезков, прямой, плоскости.

Ищешь, школьные учебники и читаешь параллельно в интернете.
389 28244
>>28237

>Учишься обнимать дроби


Ня!
390 28245
>>28237

>9.Учишься обнимать дроби


Это что такое?
391 28246
>>28237

>Учишься обнимать дроби


Лол.
392 28314
>>13314
Контингент из детишек в основном, в плане насмешек ничего не будет. Я сам поступал в 22, сейчас на втором курсе(МФТИ).
393 28325
>>28314

>Я сам поступал в 22


Для человека, которому тридцать, ты от абитуриента стандартного возраста (семнадцатилетнего школьника) в этот момент не отличался ничем.
EC5FB6D3-3F1A-4FEA-9225-08BC51B3D0E0.png1,2 Мб, 2048x1536
394 28445
>>28129

>Ну такэто же ОБЗОР, вот и куча примеров где нетрудно увидеть, очевидно, легко понять что.


Он же пишет не «легко видеть», а «отсюда следует».

>Помню как Хеллер рыдал от этой книги.


Хеллер нуб, поэтому.

Мне кажется, или это вообще очевидно неверно?
395 28454
>>28445
Легко видеть, что отсюда следует.
4ABE72E9-4C5D-4A7D-AE9D-D5344CAD0F87.png1,1 Мб, 2048x1536
396 28755
Шестимерное расслаивается над четырёхмерным, слои должны быть двумерными, нет?
397 28965
>>28314
К чему это было сказано? Спрашивал человек 24х лет.
398 28966
>>28965
Пока соберётся, уже 30 стукнет.
399 29258
/sci/ форчана скатился в трэш про мемы и брэйнлетов и притягивает щитпостеров и научпоп как магнит
/sci/ двача аналогично, все хотят понимать кванты по статьям из ленты и википедии
/math/ тут тоже мем на меме, в итоге обсуждать ничего не получается, да и знающих людей удивительно мало
На dxdy/stackexchange дышать свободно нельзя, Physicsforums/reddit получше, но и ими я уже наелся.

В последнее время мне просто становится жалко тратить время на форумы, когда я могу книгу полезную полистать. Кризис среднего возраста? Зачем меня тянет помогать другим на форумах, когда мне никто ни в чём не помогал и я смог всё понять сам?
400 29269
>>29258

>мне никто ни в чём не помогал и я смог всё понять сам


Оттого и тянет, вестимо.
401 29280
>>29258
Дико двачую брейнлетов, пиздец заебали. А вообще, добро пожаловать в постмодернизм.
402 29420
>>29258
Кстати, мне тут подумалось... а вна фриноде есть соответствующие (живые) каналы?
403 29588
>>29280
Форчан поначалу кажется забавным и продвинутым, потом от него хочется блевать дальше, чем видишь.
404 29602
>>29280
Годнота, надо здесь зафорсить.
405 30190
Хелп, плиз.

Волны Де Бройля.
"В соответствии с принятой терминологией говорят, что волны де Бройля связаны с любыми частицами и отражают их волновую природу."

У нас есть неподвижные объект - кристаллическая решётка, в которую жёстко заделаны атомы. Какова будет длина волны Де Бройля для этих атомов? Как искать?
406 30279
>>29258
а что значит дышать свободно нельзя?
407 30280
>>30279
Не дают спокойно порассуждать о вечных двигателях и моем видении квантовой механики. Хотя, physics.steckexchange близок к этому.
5.png155 Кб, 935x523
408 30292
Аноны, помогите плис не понимаю почему
данная сумма равна разности сум, верхний предел суммирования которых равны n и m соответсвенно.
409 30537
>>30292
Нарисуй на бумажке ось, на ней отметь 1, n, m (m<n из условия)
Условно обведи части оси, которые соответствуют каждой из трёх сумм
410 30648
Я вот смотрю на "координатный метод", на идею о том, что точки в пространствах можно представлять в алгебраическом виде, порождать всякими уравнениями совокупности точек, которые можно считать геометрическими объектами. Потом заглядываю в манифест Дьедонне. Потом на школьную геометрию. И возникает вопрос: есть ли смысл нубу в геометрии перечитывать школьные учебники? Что-нибудь полезное там есть?
411 30653
>>30648
Ну, формулы площадей, объёмов.
Свойства квадратов, параллелеграммов, трапеций, ромбов, конусов
Свойства n-угольников, окружностей.
Свойства сфер, цилиндров, конусов, пирамид.
Свойства отрезков, плоскостей, пространства.
Свойства препендекуляров, параллельных прямых, точек.
Свойства всяких тетраэдров, октаэдров, гексаэдров, икосаэдров, додекаэдров.
Свойства кубов, параллелепипедов, призм.
Свойства векторов, компларности, коллинеарность, тригонометрическая интерпретация векторов.
Тригонометрические функции, их свойства, их связь с формами и фигурами.

Школьная геометрия харкдорнее, слышал, харкдорнее первого курса вузов.
Вся школьная геометрия, состоит не просто из списка свойств форм пространства, а в сути знания того, как малая часть информации о форме, определяет остальную.
412 30654
>>30648

>И возникает вопрос: есть ли смысл нубу в геометрии перечитывать школьные учебники? Что-нибудь полезное там есть?


Почитай Берже Геометрия, там будет школьная геометрия через алгебру.
413 30655
>>30654
Лютый харкдор.
Эти книги читают Боги?
 .png363 Кб, 935x960
414 30679
Что это, blyatt, такое и как это решается?
Снимок.PNG1 Кб, 284x46
415 31239
Шалом, в треде для начинающих срет какой-то шизик, так что продублирую тут.
Даун-аутист в треде.
Как найти дифференциал в точке, конечно этой хуйни?
416 31249
>>31239
Не математика
417 31252
>>31249
Ты и тут уже, блядь!
418 31257
>>31239
Что такое дифференциал?
419 31278
>>31257
dx, сорт оф производная, как я понял.
420 31285
>>31257
Оператор в комплексе де Рама. Нильпотентен по умножению со степенью 2.
421 31286
Рассмотрим комплекс:
... —> C(n−1) —> C(n) —> C(n+1) —> ...
где C это дифференциальные формы — то есть элементы алгебры, порожденной символами dx(1), ... dx(n) с соотношениями
(dx)^2 = 0,
dx1 dx2 = - dx1 dx2.
Называется он комплексом де Рама.
Отображение d(n) называется дифференциалом.
422 31287
>>31286
dx1 dx2 = – dx2 dx1
Опечатка. Дифф форма это антисимметрический тензор. Элемент алгебры Грассмана, иначе.
423 31484
Хотел бы понять, как это решается. Первые три еще понятно, где-то сокращенное умножение, где-то дискриминанта, но дальше плохо понимаю. Может кто-нибудь обьяснить, как это решать? Не готовый ответ, а именно как это делается.
IMG1812.JPG2 Мб, 3024x4032
424 31487
425 31625
>>31487
(x3-1)/(x-1) тупо делится

sin(4x)/x = 4 sin(4x)/4x = 1 замечательный предел

(корень(1+x)-1)/x
домножить по формуле (a-b)(a+b)=a2-b2
x/(x(корень(1+x)+1)) = 1/(корень(1+x)+1) = 1/2
426 31751
Не сходящаяся последовательность = нефундаментальная последовательность? Ведь любая сходящаяся последовательность фундаментальна.
427 31755
>>31751
Да. Если последовательность фундоментальная, то она сходится.
428 31756
>>31755

>Если последовательность фундоментальная, то она сходится


>Каждая сходящаяся последовательность является фундаментальной, но не каждая фундаментальная последовательность сходится

429 31768
>>31756
Пополнил тебе за щеку и всё сошлось.
430 31782
>>31755
>>31756
Ну, мне препод на паре давал задания, одно из них было написать определение не сходящейся последовательности. Я написал определение нефундаментальной. Это ведь тоже верно?
431 31783
>>31782
В неполных пространствах бывают фундаментальные несходящиеся последовательности. То есть если ты доказал, что последовательность несходящаяся, это ещё не значит, что она нефундаментальная. Определения несходящейся и нефундаментальной поэтому не равносильны.
432 31784
>>31782
То есть спорный случай. Я бы поставил плюс-минус с объяснением, нуль бы ставить не стал. Как поступит твой препод - хз.
433 31786
>>31782
Верно. Если последовательность нефундаментальна, то она не сходится.
434 31787
>>31786
а фундаментальные последовательности не обязаны сходится.
Как пример последовательность приближений sqrt(2) в Q фундаментальна, но не сходится.
435 31883
Расскажите за ваше отношение к компуцкерам?
Вы приверженец высокопроизводительных мощных железок или адепт пера и бумаги ну и вообще в целом, как работаете, какие программы юзаете
436 31983
>>31883

>работаете


>/math

437 32471
>>31787
Как пример последовательность приближений sqrt(2) в Q фундаментальна, но не сходится.

Это с чего бы вдруг она не сходится?
438 32481
>>32471
Корень из двух - иррациональное число, не является элементом Q.
439 32539
>>32481
Это значит лишь то, что ее предел не принадлежит Q, а не то, что она не сходится.
440 32540
>>32481
Это значит лишь то, что ее предел не принадлежит Q, а не то, что она не сходится.
441 32559
>>32539
Любая фундаментальная последовательность точек метрического пространства сходится в пополнении этого пространства. Ты реально не понимаешь?
442 32569
>>32559
Я понимаю, что последовательнсть приближений корня из двух сходится, а ее предел не лежит в Q, ты реально этого не понимаешь?
image.png19 Кб, 1234x92
443 32938
Переотправляю свой вопрос из другого треда для начинающих.
Правда ли, что векторное произведение главных векторов плоскостей задаёт направляющий вектор прямой, по которой эти плоскости пересекаются, при любом базисе пространства? Если нет, то для чего вообще нам может пригодится этот главный вектор в неортогональном базисе?
444 32973
>>32938
Вроде да.
oVCIJgQjIpo.jpg165 Кб, 1080x1080
445 32984
не знаю, где спросить, поэтому спрошу здесь. у меня немного модифицированная задача о ранце на плоскости

в общем, есть у меня какое-то множество плоских ранцев стандартного размера. и есть какое-то множество прямоугольных (возможно квадратных) плоских вещей, которое мне нужно упаковать в плоские ранцы наиболее эффективно, то есть, таким образом, чтобы занять как можно меньшее число ранцев как можно большим числом вещей. вещи, само собой, не превышают габариты ранца ни по одной из осей

поясню смысл задачи. мне нужно нарезать из листов фанеры нарезать определённое число деталей, из которых я соберу мебель. я при этом хочу сэкономить, то есть, купить как можно меньшее число листов фанеры

какой есть математический инструментарий для решения моей задачи? я бы попробовал применить задачу о ранце, но у меня ранцев несколько
446 34452
/r книжку по комбинаторике, только чтоб совсем для даунов, с красивыми картинками и все такое.
447 34459
>>34452
Виленкин Комбинаторика.
Теория чисел sage 448 34469
Посоветуйте плез книгу (учебник) по теории чисел для школьника-старшеклассника / студента .
систематический свёрточный кодер SSk 449 34856
Сап. Возникли вопросы с ССК. Прошу помочь.
Имеется систематический свёрточный кодер (пик1) R=1/2;
Входные двоичные информационные символы поступают в сдвигающий регистр, состоящий из трех ячеек, находившийся в исходном нулевом состоянии.
После прихода на вход сдвигающего регистра очередного информационного символа коммутатор опрашивает два выхода и формирует тем самым два выходных кодовых символа.
-------
Как я понимаю, все возможные состояния такие:
000 -> 00
001 -> 01
010 -> 00
011 -> 01
100 -> 11
101 -> 10
110 -> 11
111 -> 10
-------
Однако в книге приводится кодовое дерево для данного кодера (пик2) (красным отметил первые расхождения, коих намного больше).
Мне кажется, что приведенное кодовое дерево нарушает тезис: " В случае систематического сверточного кода первым из выходных кодовых символов, получаемых за каждый цикл опроса коммутатора, всегда будет очередной информационный символ, поступивший в сдвигающий регистр."
Может я что-то неправильно понимаю. Хелп
image.png13 Кб, 477x145
450 34857
>>34856
Схема кодера
I0gb84GUkcc.jpg164 Кб, 720x960
451 35339
Здарова двач.
Короче у меня ниебецки бомбит пукан из-за говно-шараги в моей мухосрани.
Сегодня был экзамен (термех),там 3 вопроса.
1.1.Теорема о движении центра масс механической системы
Мой ответ: движение механической системы можно представить в виде движения материальной точки(центр масс) в которой сосредоточена вся масса этой системы
1.2.Основной закон динамики
Мой ответ :по второму закону Ньютона - mw=F,и в проекциях расписал .
2.1.Задача(с пика)
расписал через
mw=-cx+F(вынужд)
делил на m и заменял k^2=c/m, F представил через Hsinpt получилось:
-ap^2sin(pt)+k^2sin(pt)=Hsin(pt)
сократил на sin(pt) получил H
дальше X(вын)=H^2/(k^2-p^2)sin(pt)
подставил получил ответ
X(вын)=0.392/(98-49)
sin(7t)
правильно решено?если да то пиздец как у меня бомбит из-за "квалефекации" преподов (а педивикия и учебник Тарга говорят что в вопросах по-крайней мере я прав)
452 35340
>>35339
*H без квадрата в формулы для x
453 35636
Матемач, мне до усраки нужно решить несколько заданий по методу Стьюдента. Готов оплатить решение анона.
Пишите, кто может, на почту okkd`:e.sdlkANUSbbh9kPUNCTUMr=Knu
454 35958
двач, меня отчислить хотят. знаю, это моя проблема, но мб найдется анон что поможет мне с данной хуетой.я бы сам разобрался, но я сейчас занят другими предметами, этим займусь в последнюю очередь. заранее спасибо

3 задание на обоих листах
455 35996
Анон, изучаю книжечки из фака уровень школьника. Как эти процессы запаррелить? В школе всё было естественно - мы плыли по течению школьного курса. А теперь надо как-то самому думать. Карочи, есть какие-то гайды по последовательности изучения курса?
456 36055
Школьник 11 класс, хочу заниматься математикой, с чего начать, если у меня базовые школьные знания(не знаю как оценить объективно свой уровень, но я тот, у кого все списывают в региональной школе, думаю вы меня поняли)
2707309110792657622111154304350125271651711n.jpg140 Кб, 960x539
457 36192
Матемач, поясни мне геометрическую интерпретацию этой формулы
458 36193
>>36192
проекция непонятной хрени справа на единичный вектор, перпендикулярный двум векторам

почерк - херня полная
459 36196
>>36193
Это формула расстояния между срещивающимися прямыми
460 36207
>>35996
Не знаю , что ты там читаешь, но выстроить последовательность можно взглянув на содержание книг. Если не получится, то выручат школьные учебники.
461 36226
>>36207
Хорошо, спасибо. Как думаешь, я добьюсь успеха? :3
200px-Ato[1].jpg8 Кб, 200x162
462 36251
463 36391
Как решать подобные задачи (нужно подобрать а, чтобы было одно решение)?
Ответ тут 1, но нашел я его в проге, а не сам.
2733269115478168220012567698159868073021537n.jpg168 Кб, 406x662
464 36416
Матемач, помоги мне разобраться как решить эту залупу . В первую очередь не понимаю как решить (0-)^(-3). Что за ебучий 0-?
sage 465 36417
>>36416
ответ : ноль,вроде очевидно просто..
466 36418
>>36417
А объяснить можешь, пожалуйста?
467 36422
>>36416

>Что за ебучий 0-?


Это надо спрашивать у семинариста/лектора, ибо я бы за такие обозначения ебал бы в сраку. Попробую сванговать: 0- обозначает, что к нулю подходят со стороны отрицательных чисел, т.е слева, аналогично 0+ обозначает, что к нулю подходим со стороны положительных чисел.
468 36424
>>36418
он врёт.
Минус бесконечность.
Видно по графику функции y=1/(x^3).
Это ты в какой стране евросоюза? Ты что там делаешь? Ты что, ебанутый?
469 36429
>>36424
В пшекостане. Да я ебанутый
470 36449
>>36429
Тши пан теж ту мешкайон?

Как там по сравнению с ДС эрафии?
471 36569
>>36449
По сравнению с ДС залупа, но в целом довольно миленько и преподы не уебаны
472 36708
Привет! о/
Начинающий, на досуге от подготовки к экзаменам недавно начал читать аллуфи. Мб у кого-нибудь есть желание как-нибудь порешать задачки из него или откуда-нибудь еще? В принципе можно по любому разделу, но сейчас больше угараю по алгебре.
473 36709
Нужны книги, имеющие следующую структуру: пара-тройка страниц текста - пара-тройка упражнений (желательно с решебником или хотя бы ответами в конце). Из подобного могу назвать курс по исчислению Куранта и курс анализа Шилова. Можно на английском/немецком.
474 36710
>>36709
Напиши сам такую
475 36721
>>36709
Прасолов Задачи и теоремы линейной алгебры.
http://prasolov.loegria.net/linalg.pdf
476 36739
>>36708
Каждые пару месяцев стабильно появляется такой желающий почитать аллуфи - и столь же стабильно через недельку пропадает. Попробуй, может у тебя получится лучше остальных.
477 36742
>>36739
он дико нудный, а ещё фамильярный донельзя, от чего становится ещё нуднее. ему не хватает раскрасок только к тексту, такой там стиль блять

в своё время тоже поддался на рой бесчисленных рекомендаций читать эту книгу

Ван дер Варден - вот классный учебник алгебры, я считаю
478 36772
>>36739

>Попробуй, может у тебя получится лучше остальных.


Тут надо брать какой-то учебник с полным решебником и так проходить, сверяя свои решения. А так почитывать на досуге - просто терять свое время.
uoXrBpgAOk.jpg41 Кб, 1920x94
479 36875
помогите решить задание по топологии,без него не видать мне зачета
1261496199.jpg45 Кб, 410x280
480 36877
Привет завсегдатые. Воощем, учусь на первом курсе ФИЗФАКА МухГУ, естественно преподам поебать, в качестве оттачивания навыков дают по одному номеру каких-нибудь матриц. Не унывая, решил изучать самостоятельно, только вот, столкнулся с проблемой. Я не знаю с чего начать, пробую со Сканави (в школе не решал ничего), идет тяжеловато. Собственно вопрос, как можно отточить навык решения задач, что математических, что физических? На каких задачниках?
Вроде читаю теорию, понимаю, а задачи всё равно не даются.
Извиняюсь, если мой вопрос мог показаться ебанистически тупым для кого-то.
481 36882
>>36877
Берёшь вот это
482 36883
>>36882
Берёшь вот это
http://www.mcnmo.ru/free-books/pdf/alekseev.pdf

А Сканави выкидываешь нахуй
483 36884
>>36877
реально похожи
484 36893
>>36883
Эту книгу рекомендуют те, кто её не читал.
В ней полезна только первая глава, про группы.
485 36894
>>9338 (OP)
Тема безнадёжная, но всё же. Если у кого-то вдруг, есть желание побыть сэнпаем, то я бы с великим удовольствием вслушивался в твои премудрости. Сам начиная с дробей хочу дойти до глубоких глубин и готов этому отдать себя полностью, потому что других интересов в жизни у меня не осталось.
486 36895
>>36894
А ты симпатичная?
487 36903
>>36895
Чтобы я тогда делала тут, если бы была симпатичной.
488 36904

>В ней полезна только первая глава, про группы.



Даже это лучше, чем Сканави помноженный Демидович
15190438990190.png331 Кб, 746x2115
489 36906
490 36917
>>36894
Асечку-писечкц
491 36919
>>36904
Никто и не спорит.
Сканави-Демидовичи это для олимпиадоскама. Математики там нет.
492 36925
>>36917
И это математики? Говно какое-то.
493 36926
>>36925
Согласен.
494 37002
>>36894
Что тебе мешает читать учебник и решать задачи, пидарас?
495 37003
>>37002
лучший совет и я ему следую Никольский С.М., Потапов М.К., Решетников Н.Н., Шевкин А.В. Алгебра, вечером читаю "Что такое математика" Куранта изо дня в день. Чего же мне тогда не хватает? Общения с человеком который разделял бы мои интересы. Но как и писал выше, тема безнадёжная.
496 37004
>>37002
Ах да, я не пидорас, ты ошибся.
497 37008
>>37003
Здесь можешь обсудить всё, что хочешь.
498 37024
>>37003

>разделял бы мои интересы


Это какие интересы?
499 37091
>>37024
Да всё, это ошибка и моя слабость. Извините чуваки, не особо верил в себя. Пока что действительно иду норм, не забиваю и не ленюсь, что для меня удивительно. Даже наращиваю темп усвоения материала. Стали сниться сны с задачками.
Со стороны смешно, но мне за себя впервые приятно. Есть ощущение, что я прикасаюсь к чему-то невероятно охуительному, хотя бы потому, насколько я часто встречаюсь с бесконечностью в примерах, а ведь это только начало!11
А в целом, мне один знакомый кинул мысль, что математика - это язык на котором написана реальность. Для столь потерянного человека как я, подобное изречение стало откровением. И вот, так я попал сюда... потом подумал, что может, есть ещё такие неудачники, вроде меня, но убедился, что вы парни серьёзные.
500 37095
>>37091

>язык на котором написана реальность.


Не надо смешивать реальность и математику. А сны это хорошо, это значит ты впускаешь в себя, проникаешься ими.
image.jpeg4,7 Мб, 3264x2448
501 37173
Повторю свой реквест: планирую начать со Сканави, потом Винберг и Зорич.

В след году планирую поступить на второе высшее (теорфиз)

Опытные аноны, оцените
image.png1,5 Мб, 1000x1000
502 37223
Не знаю какой тред эффективнее, поэтому попытаюсь и здесь.

В общем, я школьник-самоучка, который угорел по математике. И я недавно видел, как можно представить функцию 1/(1-x) в виде суммы 1+x+x^2+x^3... и т.д. Но этот ряд работает только для х по модулю меньше 1. Меня это не устроило, и я "открыл" другой ряд который описывает эту функцию только модуль у числа должен быть больше 1. Ну так вот, мне стало интересно кто и когда до меня уже это открыл. Можете подсказать какой-нибудь сайт или как искать те или иные математические открытия?
503 37225
>>37223
Посмотри ряд Тейлора, по идее вопросы должны отпасть.
504 37226
>>37225
Ряд Тейлора даёт возможность найти ряд который действует в пределах от -1 до 1(не включительно), а я нашёл в пределах (-∞;-1)U(1;+∞). Мне в этом вопросе не совсем интересен ответ (я его и так уже знаю) , меня интересует существование какого-нибудь портала с архивами подобных математический открытий.
505 37288
>>37173
Теорфиз это прикмат? Тогда зачем тебе алгебра? Зорич пойдет, но если окажется сложно (учитывая, что ты начинаешь с "элементарной математики"), можешь попробовать Кудрявцева или вообще книгу по калькулюсу навернуть. Уверен, тебе так же пригодилась бы линейная алгебра, так что замени Винберга на что-то из соответствующего раздела шапки общего треда.
506 37295
>>37226
Чому это ряд Тейлора не дает возможности найти ряд от 1 до inf? Главное чтобы производная была
507 37314
>>37288
Начал с Дискретной Математики, Хаггарти.

Линал буду изучать после уже, по книге Позняка.
508 37582
Помогите, пожалуйста, придумать монотонную функцию с конечным множеством точек разрыва
509 37610
>>37582
Ты серьезно?
510 37856
Сап, матаны. Посоветуйте хорошую литературу по математическому моделированию, желательно не слишком сложную.
Снимок.PNG46 Кб, 783x318
511 37897
Пасаны, начал повторять матрицы. запнулся на блочных матрицах. Я не совсем понимаю, как выводится правило умножения блочных матриц. Вот смотрите:

http://life-prog.ru/1_36454_blochnie-matritsi.html

в конце есть лихой переход от 1 к 2 пикрилейтед. Объясните доступным языком чисто с формальной точки зрения, как это и почему это верно
512 38089
Сап, добрач. Если бесконечность - это бесконечное множество, которых существует, в свою очередь, бесконечное количество, одно больше другого, то что может быть больше Абсолютной бесконечности, понятие которой ввел Кантор? Больше множеств классы, а что больше классов? И как далеко можно зайти, если следовать по такому индуктивному пути - пути увеличения масштаба? Упремся ли мы во что-то всеобъемлющее, типа Абсолютного Всего/множества всех множеств/метаконцепции/Универсума/Универсальнокласса-всех-классов, включающий в себя любой гипотетически объектневажно, находится ли он в пределах нашего понимания, или нет, но учитывающее все парадоксы, или увеличивать масштаб можно будет бесконечно? В /б/ послали, на фочане тоже, выручайте, братцы, есть и спать не могу, хочу узнать ответ. Спасибо!
513 38149
>>37173
а первое высшее какое
514 38634
>>38089
Абстракция супермножества смысла не имеет.
515 38767
Пытаюсь учить доказательства. В учебнике задача:

Suppose a is an integer. If 7|4a, then 7|a.


У меня решение такое:

>Proof. Suppose 7|4a. Then 4a = 7x. a = 7x/4; a = 7 * (x / 4). Suppose (x / 4) = 4; Then a = 7d. By definition a is divisible by 7.



Но в учебнике решение этого примера такое:

>Proof. Suppose7|4a.


By definition of divisibility, this means 4a = 7c for some integer c.
Since 4a = 2(2a) it follows that 4a is even, and since 4a = 7c, we know 7c is even. But then c can’t be odd, because that would make 7c odd, not even.
Thus c is even, so c = 2d for some integer d.
Now go back to the equation 4a=7c and plug in c=2d. We get 4a=14d. Dividing both sides by 2 gives 2a = 7d.
Now, since 2a = 7d, it follows that 7d is even, and thus d cannot be odd.
Then d is even, so d = 2e for some integer e.
Plugging d = 2e back into 2a = 7d gives 2a = 14e.
Dividing both sides of 2a = 14e by 2 produces a = 7e.
Finally, the equation a = 7e means that 7 | a, by definition of divisibility.

Верно ли мое решение, и если нет, то почему?
516 38768
>>38767
опечатка

> Suppose (x / 4) = 4


> Suppose (x / 4) = d

517 39002
>>38634
Почему, анон?
производная.jpg23 Кб, 590x190
518 39095
Привет, математики. Мне нужно не решение, с ним я и сам справляюсь, а подсказка. Есть функция, надо найти производную. И что-то я безбожно туплю - как относиться в ней к a? Как к константе или переменной? Или я даже не в ту сторону думаю? Сразу скажу, что отучился давно и могу чего-то действительно не помнить, по дружбе помогаю решить пару задачек. В общем, как брать производную конкретно от а?
519 39272
>>39095
Как к константе.
Math.JPG18 Кб, 481x154
520 39305
Сап, математики, буду благодарен если поможете мне с этой задачей, буду благодарен любой помощи:

Компания производит два вида полимерных материалов: полипропилен и полистирол. Для этого она нанимает неквалифицированных, квалифицированных и высококвалифицированных рабочих. Почасовая оплата в у.е. каждого рабочего и количество тонн материалов, производимым каждым рабочим в час, приводятся в таблице.

Рабочие могут производить одновременно оба продукта, например, квалифицированный рабочий может производить 3 тонны полипропилена 3 тонны полистирола каждый час. Поступил заказ произвести за один час 21 тонну полипропилена и 15 тонн полистирола

Требуется определить, сколько рабочих каждой квалификации необходимо нанять, чтобы выполнить заказ с наименьшими затратами на оплату труда. Постройте математическую модель и найдите оптимальное решение.
17315240.jpg32 Кб, 620x444
521 39915
Ребят, учусь на программиста в вузике и матанализ - это самое сложное на 1 курсе. Мало того, что препод пытается нам в головы впихнуть то, что сам выучил на прикладмате за 3 года за год, так ещё и времени нету сидеть подолгу за ним. Изучение матанализа для меня превратилось в сплошную боль, я даже формулу Тейлора банальную с нескольких попыток так и не прошарил по-человечески. Так вот: я уже долго ищу какую-нибудь книгу по матанализу типа "Матанализ для дибилов" где всё объяснено человеческим языком, а не сухими, тяжело перевариваемыми формулировками. Если кому-то из вас известны материалы (очень желательна/ны именно книга/и а не видео) то поделитемь пожалуйста со мной!
uoXrBpgAOk.jpg41 Кб, 1920x94
522 40098
помогите,осталось 1 задание чтобы сдать топологию
523 40197
>>38149

Юриспруденция.
Безымян232ный.png6 Кб, 308x88
524 40230
Хелпуй ананас. Решаю Сканави потихоньку, и сталкиваюсь с таким примером как на фото. Проблема в том что я не могу понять нужно ли включать число 2 в решение или нет. тк при х=2 ответ ноль, при х принадлежащем отрезку(2;+бесконечность)-ответ корень из икс, и при х принадлежащем отрезку(0;2) - ответ -1корень из х. Ну так писать ли мне что при х = 2 ответ ноль или хватит и того что при х принадлежащем отрезку(2;+бесконечность)-ответ корень из икс, и при х принадлежащем отрезку(0;2) - ответ -1корень из х? в ответах к сканави число 2 не включено.
525 40412
Аноны, с чего начать вкатывание? Сдал на днях последние егэ, думаю теперь заняться нормальной математикой (для себя и чтобы быть готовым к вузу)
526 40982
>>40412
Бамп реквесту и треду.
Посоветуйте, с каких книг закатиться в выш. мат. Пределы, интегралы, диф. уравнения, дискретная математика.
527 41202
Докажите, что касательная к окружности, описанной около
неравнобедренного треугольника, в его вершине пересекает продол-
жение противоположной стороны в середине отрезка, концы которого
совпадают с основаниями биссектрис внутреннего и внешнего углов
при этой вершине. (Не знаю с чего и как начать доказательство, подсказка не особо помогла. Помогите пожалуйста)
528 41206
>>41202
И не подскажите,что нибудь серьезное для прокачивание скилла на доказательства теорем(проблем с использованием матана нет,но доказательства порой тяжко даются)
529 41293
Год мечтаю и пытаюсь понять, но не понял. Речь о репертуарном методе Кнута в Конкретной математике.
В общем, дано рекуррентное соотношение общего вида ф(1)=а, ф(2н)=2(ф(н))+в, ф(2н+1)=2(ф(н))+с.
Автор замечает, что сие суть ф(н)=А(н)а+В(н)в+С(н)с.
Так как это общего вида формула, он начинает подбирать нужные а, в, с, заменять ф(н) на всякие другие в рамках вышеописанных формул. Получал он при этом ... вот тут у меня бомбит хлеще, чем от того, что написал выше, ибо я слоупок. Так что он получал? Систему чего? Вот ах+bу+сz=д я ещё переживу, но у автора стоят различные неизвестные функции от н. Где это читать, учить, понять? В линейной алгебре не увидел похожего. Анонимно, ты ведь умен как Мотидзуки! Помоги!
530 41866
>>41293
Иногда подбором.
531 42005
>>9338 (OP)
Тыкните носом где тут по статистике угорают?
532 42160
>>42005
Вот по прикладной матёше: https://2ch.hk/math/res/19499.html (М)
Отдельный тред по статистике можешь сам создать.
533 42181
>>26416
Берёшь листочки матшколы и наслаждаешься.
534 42226
Как учить математику если ты тупой и ничего не знаешь?
535 42233
С чего начать изучение математики?
536 42234
>>42226
>>42233
Сидеть и решать задачи, потом проверять себя. Продолжать это вечно.
537 42235
>>42234
Не, я имею в виду, по какой литературе заниматься? Хочется получить основательное понимание и математическую базу, чтобы потом это не мешало изучать физику, например. Знать хотя бы на вузовском уровне.
538 42238
>>42235
Вузовский уровень физики это что? Общефиз или теорфиз? Если первое, то достаточно взять Берклеевский курс и Фейнмановские лекции с задачником к нему. В лекциях там нужная математика разжёвывается.
539 42249
>>42234
Будто можно просто взять и решать задачи. Надо еще знать откуда и какие...
540 42326
А почему бы и не перекатить тред?
541 42406
>>42226
Во время занятий математикой очень важен правильный настрой. Математика любит инициативу, настойчивость и эмоциональную стабильность. А все вот эти мысли - я тупой, ничего не умею, и т.п. - не объективная истина, а, как правило, мнение о вас родителей, училок и прочих биологических отходов, желающих самоутвердиться за ваш счет. С другой стороны, даже профессионалы порой испытывают фрустрацию или раздражение, когда у них не получается какая-то сложная задача. Тогда есть смысл отложить одну задачу и заняться пока другой. Вообще советую погуглить "Правила В.Ю.", это какой-то московский репетитор, составивший список "установок" на продуктивные занятия. Хорошо написал и по делу.
542 44050
Прикрепленный тред на доске направил меня сюда. А в этом треде я расчитывал увидеть FAQ или картиночку с канонической по версии двача математической библиотекой.
543 44065
>>12210
Это украинская математика, там всё можно
1016926802.jpg52 Кб, 500x670
544 44416
Что можете сказать про эти два учебника, можно ли с них начинать закатываться человеку, который до этого не имел дело с математикой? Как они по повествованию, не слишком ли сложно для ньюфага будет?

1 - https://www.ozon.ru/context/detail/id/139242570/
2 - https://www.ozon.ru/context/detail/id/139242573/
545 44696
Хотелось бы узнать по поводу начал мат.ана.
С чего обычно начинают и трудно ли это?
546 46488
Назовём дроби a/b и c/d (a, b, c, d – целые положительные числа) «соседними», если их разность (ad – bc)/bd имеет числитель 1, то есть если
ad – bc =1.
Докажите, что в этом случае никакая дробь e/f с натуральными e и f , у которой f<b +d , не находится между a/b и c/d.

Задача из книги Гельфанда. Никак не могу подступиться. Кто подскажет /направит?
Screenshot2018-12-23-20-33-49-988com.google.android.apps.do[...].png425 Кб, 1080x1920
547 47487
Двач, помоги решить второе неравенство
548 56180
>>14446
Это лучшее, что я видел в своей жизни
549 56453
Народ,можете посоветовать учебник по геометрии,желательно с самых азов,что бы все было внятно и новичку вроде меня было легко ее изучать
изображение.png3 Кб, 221x110
550 58220
Как искать формулу степени в степени, не нашёл в свойствах степеней
551 59611
>>58220
На сколько помню там умножение, т.е в кружке 15
552 59815
>>59611
Да я уже разобралс, пока тут ответа дождалс, на самом деле там надо вычислить 5 в кубе буит 75, это же степень
image.png1,7 Мб, 1262x765
553 60281
Нужна помощь ньюфагу.
Понял, что ничего не понял и теперь хочу начать все с самого начала.
Можете посоветовать каких-нибудь убегодных книг для вкатывания в математику с постепенным повышением сложности. Вот прям с самых азов для школьников средних классов.
Важное условие - максимально простой и понятный язык, доступный даже дауну.

В школе учили решать по шаблонам, в универе то же самое. Сидишь, дрочишь дифуры, интегралы, не понимая зачем. Вроде бы даже сдаешь экзамен на 4-5, а через пару месяцев понимаешь, что мало того, что ничего не понял, так теперь еще и забыл. Из всего курса более-менее на начальном уровне понял только тервер, даже порой пригождается в жизни.
554 60430
>>60281
курсы на степике (там прям по разные есть), mathprofi.ru

https://www.youtube.com/channel/UCz1OkR6uwWeyut2eBU0CG-A

и самое главное найди приминение ей, типо нейроночки или для расчетов в R studio, matlab, весьма стимулирует. Сам жалею шо в колледже забил на матеш, просто толкали и не давали мотивации
555 60768
>>9338 (OP)
Дана функция x(t)=ln(t^2) в пространстве L2 со стандартным скалярным произведением. Для функции x(t) в этом пространстве требуется найти многочлены наилучшего приближения первой и второй степени.
Проблемы возникают на этапе вычисления коэффициентов Фурье. получается, что для первого коэффициента интеграл (1/sqrt(2)*ln(t^2)) в пределах от -1 до 1 равен минус бесконечности. С другим коэффициентами в принципе дела не лучше.
Что делать в таком случае, как дальше подставлять это в формулу для многочленов приближения?
556 60988
Ребят, простая задачка по ТЧ, школьная:
доказать, что сумма значений функции Эйлера на делителях натурального числа равна ему самому. Изи решил "в лоб", воспользовавшись мультипликативностью функции и посчитав для степени простого числа, но просят посчитать комбинаторным способом, а у меня рука на это не набита, хз чё делать, не гуглится( Помогите(
IMG20200108142456.jpg23 Кб, 540x215
557 63714
Извиняюсь за то что я такой слоупок,но дорогие аноны,вы не сможете ли подсказать как делать четвертый и пятый номер на этом бланке?С остальными номерами кроме этих двух проблем вроде нет,но здесь прям вообще пиздец какой-то,никаких идей подходящих нет.
558 64207
>>9338 (OP)
Анонасы выручайте. Вот вам дан пример, решается интеграл по методу приведения трехчлена к канонической формы. В демидовиче ответ сходится, объясняли однокурсники схожим образом, на проверке у препода подобные проверенные задания зачтены. Так вот, почему это верно, ведь у нас dx. Если бы заместо (х-1/4), стояло бы просто х, вопросов нет, используем форму и готово. Но ведь там (х-1/4) и для дальнейшего, нам нужно, например, воспользоваться методом введения под диференциал, т.е. d(x-1/4). Где я обосрался объясните анонасы? И да, почти все онлайн калькуляторы, выдают решение отличное от этого. Поэтому и еще больше сомнений.
559 64211
>>64207
Уже дали ответ, не отвечайте анонасы.
560 64254
Привет. Друзья, такая ситуация. Когда-то давно дропнул техвуз, сейчас работаю программистом. Не так давно открыл для себя НМУ, просто в свободное время смотрел их лекции и решал задачки. В итоге не заметил, как прошёл первый семестр - послушал всё, что было и сделал большую часть задач.
И собственно вопросик вот в чём. У меня же всё ок получается, математика такого сорта (чистая, если хотите) меня всегда привлекала гораздо сильнее, чем любая другая область вообще. Другая область - вообще что угодно: рисование, музыка, спорт, етц.. Ничего прекраснее и увлекательнее лично для себя не нашёл. И вот раз это у меня получается, раз это так нравится, но тем не менее мне уже около 24 годиков, то стоит ли пытаться поступить туда по-нормальному, и будет ли шанс после окончания пойти в аспу какого-нибудь вузика и заняться нормальной научной деятельностью? Или всё оставить так, как есть - прогать (что тоже нравится) и продолжать заниматься в качестве хобби, не делая на учебу особых акцентов?
561 65384
Привет всем математикам.
Мне 19-2 (возможно) лет и мне тяжело смотреть в сторону математики. В школе мне попались больные учителя, которые не могли нормально преподавать предмет, делали это безынтересно и вызывали отвращение. Да и сам я что сейчас что раньше был мягко говоря не сахар. Итак, вопрос. Могу ли "полюбить" матан? Или уже всё запущено возрастом, мозгоёблей?

Почему задался вопросом? Искренне верю что лишь знание математики и логики на должном уровне сможет сделать меня нормальным погромистом (300к/нс), а не очередным педоверстальщиком.
Кстати говоря, сейчас у меня знании математики хуевые плохие. Не даётся техникумом. Я могу заставить себя изучать тему, но от того что я ЗАСТАВЛЯЮ себя учить, становится лишь тяжелее. Может таблеточки посоветуете? Книжки? Собственные стори? Хочу вкатиться просто и не мучаясь понапрасну.
562 65671
>>28237

>Учишься обнимать дроби


И тут я капитулировал!
screenshot.67.png174 Кб, 1430x958
563 66378
Поясните престарелому бумеру, почему так можно сделать при доказательстве?
564 66961
>>65384
Либо ты получаешь удовольствие от результата твоего решения, либо нет. Все просто.
А боль в процессе решения - нормально. Мышцы знаешь ли тоже молочной(вроде) кислотой наливаются, когда физуху хуяришь.
TdegyFOWQV8.jpg86 Кб, 1080x806
565 68073
>>9338 (OP)
Я нулевый в математике, поэтому прошу, не смейтесь. Я не тролль, я просто прошу помощи, именно математику я знаю хуже всего. В общем. Научите как делить в столбик, я не понимаю, пожалуйста.
566 68077
567 70039
Есть ли какой тест, чтобы определить свой уровень знания в математике и алгебре?
568 70407
>>68073
Помню был сайт для подготовки к ЕГЭ для совсем нулёвых. Там были раздельные задания для того, чтобы сдать на 3, 4 и 5 и они разжовывались очень просто. Подскажите, кто знает.
569 70595
хочу вкатится в матан. на матиматику забил класса с 7, сейчас захотелось заняться. Подскажите как вкатится, в каком порядке изучать и где, пожалуйста
570 70857
>>9338 (OP)
Сап, математач. Я излазил гугл вдоль и поперёк, но так и не нашёл никаких формул или внятных объяснений тому, как делается то, что требуется в задании с пикрила1. Помогите, умоляю.
d=40, h=50, k(x,y,z)=(30,30,5), a(x,z)=(39,19), b(z)=(19).
571 71483
Сяп, скрытая доска. Существует ли ещё способы показать (не)изоморфизм графов помимо нахождение дополнительных графов?
Да и если порядок вершин не определен, а изначальное отображение не установлено, то как тогда всё это дело доказывается? Читаю Белоусова, у него там только дополнительные графы (судя по всему для очень плотных графов), а так просто индуктивно прослеживается соответсвие вершин и ребёр.
572 71682
Слушайте, я чет не нашел ни одной ссылки на вменяемые каналы. Кто-то может что-то посоветовать? Где достаточно полно освещают школьную программу и на хорошем уровне универскую? Я понимаю, что в ютубе много лекций и если выборочно искать, то найти можно, но хочется чтобы все было в одном месте более менее. Посоветуйте.
573 72160
Сап добропостач. У меня истерика случилась по поводу задач на растворы/смеси/сплавы/движение/проценты и прочее.
Теперь я хочу научиться их решать настолько хорошо, чтобы быть подобно тем людям, которых в школ гнобили и унижали, а теперь они сами кого хочешь унизят (только в плане задач). Поэтому реквестирую задачники, на которых можно натренироваться.

Я шкила, только в 10 класс перешёл. Поэтому если каким-то чудом у вас будут материалы для 7-9 буду искренне благодарен. С меня как всегда.
574 72389
Аноны, какие самые годные русскоязычные учебники по дискретной математике вы знаете и какой минимум нужен для его освоения?
575 74906
>>9338 (OP)

>Привет! У меня высшего мат.образования, поэтому прошу гнилые помидоры не кидать - пишу в треде для начинающих.



Короче, вот такой текст в учебнике:
Замечание (Внимание! Только для особо интересующихся).
Пусть: A — утверждение, «x» — переменная, «ϕ» — выражение.
Возникает впечатление, что справедливо следующее правило:
для построения утверждения Subst(A; x;ϕ) нужно
в утверждении A заменить переменную «x» на выражение «ϕ».
Однако, здесь возникает целый ряд трудностей, связанных с тем, что:
переменная «x» может больше одного раза входить в утверждение A;
переменная «x» в утверждении A может находиться в области действия некоторого квантора по переменной «x»;
переменная «x» в утверждении A может находиться в области действия некоторого квантора по некоторой переменной,
содержащейся в выражении «ϕ».

Subst - подстановка

Я не понмаю, в чем именно состоит ряд трудностей

Последние два пункта - про вхождение в область кванторов - просто если, например, заменить в выражении, в котором x четное число, некоторую переменную на выражение, в котором утверждается, что х- нечетное, (и наоборот) то выражение станет бессмысленным?

Про вхождение более одного раза вообще непонятно
Ну, в Subst(P(x,x,y),x,A)->P(A,A,y)? Разве нет?
576 80181
>>9338 (OP)
Продублирую

Аноны, направьте, пожалуйста, а то я не могу нагуглить решение:
Для циклического кода даны:
Мощность кода(M) = 125
Количество информационных символов(k) = 16
Вектор ошибки х7
Количество обнаруживаемых ошибок(s) = 1
Количество исправляемых ошибок(r) = 1

Я посчитал:
Кодовое расстояние(d) = 3
Кол-во контрольных разрядов(m) = 5 (нижняя граница по оценке Хэмминга и Варшамова-Гилберта)
Порождающий полином(G(x)) = x5+x4+1
Проверочный полином(H(x)) = x16+x15+x14+x13+x12+x10+x8+x5+x4+1

Проблема:
Как из имеющихся данных найти передаваемое сообщение? По условию его тоже нужно найти. Я не могу понять, как. Это как-то связано с тем, что указанная в условии мощность кода 125 много меньше, чем 2k[=16/sup]?
577 80305
тест
[mаth]5^5[/mаth]
[mаth]sqrt(16)[/mаth]
[mаth]5^(sqrt(16) * 2)[/mаth]
[mаth]log(1024,2)[/mаth]
image.png51 Кб, 891x224
578 81981
Анонче, я шкила и я единственный, кто смог в параметры. Так вот, мне прилетает по лицу эта вещь и я окончательно ломаюсь с нее. Да, она олимпиадная, но я хочу в ней разобраться.

Анонче, сможешь ли ты мне подсказать, что я должен разобрать здесь первым?
579 81982
Анонче, большие пробелы в знании базы. Какие подводные, если начну с пикрил?
580 82888
Мой максимальный уровень - это тройные интегралы. Поверхностные уже со скрипом проходят. Есть варианты прокачаться на более высокий уровень? Если да, то порекомендуйте норм источники, где на максимально простом уровне расскажут и покажут что почем. До сего момента юзал онли http://mathprofi.ru/ В данынй момент остро стоит проблема с пониманием ТФКП
image.png6 Кб, 348x118
тригонометрия 581 83335
Как привести это говно к общему знаменателю по человечески? Типо в первой дроби по формулам привидения получается -sinx - 1, а во второй, раскрыв cos2x и подставив вместо единицы осн. триг. тождество получил 2(sinx)^2. ...и чо дальше
582 83707
Приветствую. Мне 17, закончил 10 класс. Хочу понять что такое математика, что она изучает, почему ей занимаются и интересно ли это (да, это субъективно, но я и хочу посмотреть на математику с точки зрения того, кому она интересна). Познания в математике нулевые. Посоветуйте как мне вкатиться в математику ? Именно как в науку, а не чтобы подготовиться к ЕГЭ например, хотя это тоже нужно, но тут в принципе всё понятно что делать и как. Итак, что мне прочитать/посмотреть/послушать, чтобы понять что такое математика и возможно начать её изучение полноценно в будущем ? Заранее большое спасибо.
583 87888
Я в унике пинал хуи на линале и матане, а сейчас что-то проснулось желание наверстать. Какой самый приятный и эффективный способ это сделать? Чьи лекции самые топовые и какие учебники/задачники лучше курить и как? Хочу именно с нуля вникать в своём темпе в каждую теоремку и смысол каждой формулки.
584 92052
>>83335

>


>


(cosx)^2=1-(sinx)^2=(1-sinx)(1+sinx), а в знаменателе у тебя -sinx-1=-(1+sinx), т.е. можно будет сократить на 1+sinx, исключив из корней sinx=-1.
585 94081
Сап, меня напугала куча терминов в треде по топологии, так что пишу сюда. Суть вопроса: может ли гладкое неориентируемое многообразие без края быть краем какого-то другого многообразия, если да, то всегда ли найдётся такое многообразие? В частности, является ли открытая лента мебиуса краем какого-то многообразия?
в столбик.jpg207 Кб, 1280x720
586 95879
Как называется эта хуйня?
587 96423
>>64254
У меня такой же вопрос
588 97505
>>9338 (OP)
Я вот задался вопросом. Математика на егэ это же просто детский лепет? Я такой весь из себя крутой готовился 1 (один) год и надеялся на 80+ баллов по итогу 76 и я хоть и рад был что балл для поступления я набрал но было обидно за некоторые задания. Ну так вот как по вашему я просто слаб раз не смог сделать 13 16 17 18 задания или это нормально и математическое будущее у меня есть? Матека мне нравится и я планирую грызть гранит
589 97506
>>95879
то что в уме держишь, а название кому-то нужно?
590 97508
>>97505
Почему ты не смог их решить? Времени не хватило? Переволновался? Совершил арифметические ошибки? Мало потратил времени на разбор методов решения? Или просто в принципе не понял, как решать, когда задача чуть-чуть отошла от того, что ты нарешивал?
Ответив на эти вопросы, ты наполовину ответишь на свой пост.
591 97538
>>97508
Я такие вообще никогда не встречал. Выходит я не мог выкрутиться в той ситуации а значит не особо то и смыслю в матеке
592 99714
Хочу попробовать вкатиться в эту вашу математику, но что делать если я даже за начальные классы ничего не знаю/не помню. Есть ли какие видеоуроки для совсем зеленых ньюфагов? Хочу пробежаться хотя бы по 9 классам, а там может и уровень 10- 11 кл, если вкатит это дело
593 99986
>>99714
Элементарная алгебра, Туманов
594 100013
V
image.png10 Кб, 408x107
595 100686
Поможете?
596 101035
Доброго времени суток. Пытаюсь решить неравенства а-ля x <= 100log(x) или x > 10^2x, онлайн калькуляторы выдают какие-то омеги. В контексте школьной математики это как-то решается вообще?
597 102072
Многоуважаемые матаны, спасити памахите! Нужна программа для доказательства тавтологичности формул состоящих из предикатов. Вот это вообще про это?
https://ru.wikipedia.org/wiki/%D0%97%D0%B0%D0%B4%D0%B0%D1%87%D0%B0_%D0%B2%D1%8B%D0%BF%D0%BE%D0%BB%D0%BD%D0%B8%D0%BC%D0%BE%D1%81%D1%82%D0%B8_%D1%84%D0%BE%D1%80%D0%BC%D1%83%D0%BB_%D0%B2_%D1%82%D0%B5%D0%BE%D1%80%D0%B8%D1%8F%D1%85

Поддерживаемые и активно развивающиеся решатели: Alt-Ergo, Barcelogic, Beaver, Boolector, CVC3, DPT, MathSAT, OpenSMT, SatEEn, Spear, STP, UCLID, veriT, Yices, Z3.

Какая из них попроще? Кого еще можно повопрошать на эту тему?
image.png413 Кб, 550x728
598 102433
Я вероятно вообще не по теме залетел, но тут умные люди сидят, поэтому...
Можно ли вкатываться в логику с подобной книженции, или есть что-то посодержательнее для ньюфага без каких либо знаний?
Какие знания предварительно потребуются?
599 110554
>>95879
Сначала складывешь 45 и 9, получаешь 54, и складывешь с 30. Короче говоря, ты ещё один десяток держишь в уме, 4 пишешь и один (десяток обычно не говорят слово десяток) в уме
600 110555
>>83707
Сеймы насчёт как науки
Обновить тред
« /math/В начало тредаВеб-версияНастройки
/a//b//mu//s//vg/Все доски

Скачать тред только с превьюс превью и прикрепленными файлами

Второй вариант может долго скачиваться. Файлы будут только в живых или недавно утонувших тредах.Подробнее